• Prodigy Math
  • Prodigy English

From our blog

  • Is a Premium Membership Worth It?
  • Promote a Growth Mindset
  • Help Your Child Who's Struggling with Math
  • Parent's Guide to Prodigy
  • Assessments
  • Math Curriculum Coverage
  • English Curriculum Coverage
  • Game Portal

120 Math Word Problems To Challenge Students Grades 1 to 8

no image

Written by Marcus Guido

Hey teachers! 👋

Use Prodigy to spark a love for math in your students – including when solving word problems!

  • Teaching Tools
  • Subtraction
  • Multiplication
  • Mixed operations
  • Ordering and number sense
  • Comparing and sequencing
  • Physical measurement
  • Ratios and percentages
  • Probability and data relationships

You sit at your desk, ready to put a math quiz, test or activity together. The questions flow onto the document until you hit a section for word problems.

A jolt of creativity would help. But it doesn’t come.

Whether you’re a 3rd grade teacher or an 8th grade teacher preparing students for high school, translating math concepts into real world examples can certainly be a challenge.

This resource is your jolt of creativity. It provides examples and templates of math word problems for 1st to 8th grade classes.

There are 120 examples in total.

The list of examples is supplemented by tips to create engaging and challenging math word problems.

120 Math word problems, categorized by skill

Addition word problems.

A teacher is teaching three students with a whiteboard happily.

Best for: 1st grade, 2nd grade

1. Adding to 10: Ariel was playing basketball. 1 of her shots went in the hoop. 2 of her shots did not go in the hoop. How many shots were there in total?

2. Adding to 20: Adrianna has 10 pieces of gum to share with her friends. There wasn’t enough gum for all her friends, so she went to the store to get 3 more pieces of gum. How many pieces of gum does Adrianna have now?

3. Adding to 100: Adrianna has 10 pieces of gum to share with her friends. There wasn’t enough gum for all her friends, so she went to the store and got 70 pieces of strawberry gum and 10 pieces of bubble gum. How many pieces of gum does Adrianna have now?

4. Adding Slightly over 100: The restaurant has 175 normal chairs and 20 chairs for babies. How many chairs does the restaurant have in total?

5. Adding to 1,000: How many cookies did you sell if you sold 320 chocolate cookies and 270 vanilla cookies?

6. Adding to and over 10,000: The hobby store normally sells 10,576 trading cards per month. In June, the hobby store sold 15,498 more trading cards than normal. In total, how many trading cards did the hobby store sell in June?

7. Adding 3 Numbers: Billy had 2 books at home. He went to the library to take out 2 more books. He then bought 1 book. How many books does Billy have now?

8. Adding 3 Numbers to and over 100: Ashley bought a big bag of candy. The bag had 102 blue candies, 100 red candies and 94 green candies. How many candies were there in total?

Subtraction word problems

Best for: 1st grade, second grade

9. Subtracting to 10: There were 3 pizzas in total at the pizza shop. A customer bought 1 pizza. How many pizzas are left?

10. Subtracting to 20: Your friend said she had 11 stickers. When you helped her clean her desk, she only had a total of 10 stickers. How many stickers are missing?

11. Subtracting to 100: Adrianna has 100 pieces of gum to share with her friends. When she went to the park, she shared 10 pieces of strawberry gum. When she left the park, Adrianna shared another 10 pieces of bubble gum. How many pieces of gum does Adrianna have now?

Five middle school students sitting at a row of desks playing Prodigy Math on tablets.

Practice math word problems with Prodigy Math

Join millions of teachers using Prodigy to make learning fun and differentiate instruction as they answer in-game questions, including math word problems from 1st to 8th grade!

12. Subtracting Slightly over 100: Your team scored a total of 123 points. 67 points were scored in the first half. How many were scored in the second half?

13. Subtracting to 1,000: Nathan has a big ant farm. He decided to sell some of his ants. He started with 965 ants. He sold 213. How many ants does he have now?

14. Subtracting to and over 10,000: The hobby store normally sells 10,576 trading cards per month. In July, the hobby store sold a total of 20,777 trading cards. How many more trading cards did the hobby store sell in July compared with a normal month?

15. Subtracting 3 Numbers: Charlene had a pack of 35 pencil crayons. She gave 6 to her friend Theresa. She gave 3 to her friend Mandy. How many pencil crayons does Charlene have left?

16. Subtracting 3 Numbers to and over 100: Ashley bought a big bag of candy to share with her friends. In total, there were 296 candies. She gave 105 candies to Marissa. She also gave 86 candies to Kayla. How many candies were left?

Multiplication word problems

A hand holding a pen is doing calculation on a pice of papper

Best for: 2nd grade, 3rd grade

17. Multiplying 1-Digit Integers: Adrianna needs to cut a pan of brownies into pieces. She cuts 6 even columns and 3 even rows into the pan. How many brownies does she have?

18. Multiplying 2-Digit Integers: A movie theatre has 25 rows of seats with 20 seats in each row. How many seats are there in total?

19. Multiplying Integers Ending with 0: A clothing company has 4 different kinds of sweatshirts. Each year, the company makes 60,000 of each kind of sweatshirt. How many sweatshirts does the company make each year?

20. Multiplying 3 Integers: A bricklayer stacks bricks in 2 rows, with 10 bricks in each row. On top of each row, there is a stack of 6 bricks. How many bricks are there in total?

21. Multiplying 4 Integers: Cayley earns $5 an hour by delivering newspapers. She delivers newspapers 3 days each week, for 4 hours at a time. After delivering newspapers for 8 weeks, how much money will Cayley earn?

Division word problems

Best for: 3rd grade, 4th grade, 5th grade

22. Dividing 1-Digit Integers: If you have 4 pieces of candy split evenly into 2 bags, how many pieces of candy are in each bag?

23. Dividing 2-Digit Integers: If you have 80 tickets for the fair and each ride costs 5 tickets, how many rides can you go on?

24. Dividing Numbers Ending with 0: The school has $20,000 to buy new computer equipment. If each piece of equipment costs $50, how many pieces can the school buy in total?

25. Dividing 3 Integers: Melissa buys 2 packs of tennis balls for $12 in total. All together, there are 6 tennis balls. How much does 1 pack of tennis balls cost? How much does 1 tennis ball cost?

26. Interpreting Remainders: An Italian restaurant receives a shipment of 86 veal cutlets. If it takes 3 cutlets to make a dish, how many cutlets will the restaurant have left over after making as many dishes as possible?

Mixed operations word problems

A female teacher is instructing student math on a blackboard

27. Mixing Addition and Subtraction: There are 235 books in a library. On Monday, 123 books are taken out. On Tuesday, 56 books are brought back. How many books are there now?

28. Mixing Multiplication and Division: There is a group of 10 people who are ordering pizza. If each person gets 2 slices and each pizza has 4 slices, how many pizzas should they order?

29. Mixing Multiplication, Addition and Subtraction: Lana has 2 bags with 2 marbles in each bag. Markus has 2 bags with 3 marbles in each bag. How many more marbles does Markus have?

30. Mixing Division, Addition and Subtraction: Lana has 3 bags with the same amount of marbles in them, totaling 12 marbles. Markus has 3 bags with the same amount of marbles in them, totaling 18 marbles. How many more marbles does Markus have in each bag?

Ordering and number sense word problems

31. Counting to Preview Multiplication: There are 2 chalkboards in your classroom. If each chalkboard needs 2 pieces of chalk, how many pieces do you need in total?

32. Counting to Preview Division: There are 3 chalkboards in your classroom. Each chalkboard has 2 pieces of chalk. This means there are 6 pieces of chalk in total. If you take 1 piece of chalk away from each chalkboard, how many will there be in total?

33. Composing Numbers: What number is 6 tens and 10 ones?

34. Guessing Numbers: I have a 7 in the tens place. I have an even number in the ones place. I am lower than 74. What number am I?

35. Finding the Order: In the hockey game, Mitchell scored more points than William but fewer points than Auston. Who scored the most points? Who scored the fewest points?

Fractions word problems

A student is drawing on a notebook, holding a pencil.

Best for: 3rd grade, 4th grade, 5th grade, 6th grade

36. Finding Fractions of a Group: Julia went to 10 houses on her street for Halloween. 5 of the houses gave her a chocolate bar. What fraction of houses on Julia’s street gave her a chocolate bar?

37. Finding Unit Fractions: Heather is painting a portrait of her best friend, Lisa. To make it easier, she divides the portrait into 6 equal parts. What fraction represents each part of the portrait?

38. Adding Fractions with Like Denominators: Noah walks ⅓ of a kilometre to school each day. He also walks ⅓ of a kilometre to get home after school. How many kilometres does he walk in total?

39. Subtracting Fractions with Like Denominators: Last week, Whitney counted the number of juice boxes she had for school lunches. She had ⅗ of a case. This week, it’s down to ⅕ of a case. How much of the case did Whitney drink?

40. Adding Whole Numbers and Fractions with Like Denominators: At lunchtime, an ice cream parlor served 6 ¼ scoops of chocolate ice cream, 5 ¾ scoops of vanilla and 2 ¾ scoops of strawberry. How many scoops of ice cream did the parlor serve in total?

41. Subtracting Whole Numbers and Fractions with Like Denominators: For a party, Jaime had 5 ⅓ bottles of cola for her friends to drink. She drank ⅓ of a bottle herself. Her friends drank 3 ⅓. How many bottles of cola does Jaime have left?

42. Adding Fractions with Unlike Denominators: Kevin completed ½ of an assignment at school. When he was home that evening, he completed ⅚ of another assignment. How many assignments did Kevin complete?

43. Subtracting Fractions with Unlike Denominators: Packing school lunches for her kids, Patty used ⅞ of a package of ham. She also used ½ of a package of turkey. How much more ham than turkey did Patty use?

44. Multiplying Fractions: During gym class on Wednesday, the students ran for ¼ of a kilometre. On Thursday, they ran ½ as many kilometres as on Wednesday. How many kilometres did the students run on Thursday? Write your answer as a fraction.

45. Dividing Fractions: A clothing manufacturer uses ⅕ of a bottle of colour dye to make one pair of pants. The manufacturer used ⅘ of a bottle yesterday. How many pairs of pants did the manufacturer make?

46. Multiplying Fractions with Whole Numbers: Mark drank ⅚ of a carton of milk this week. Frank drank 7 times more milk than Mark. How many cartons of milk did Frank drink? Write your answer as a fraction, or as a whole or mixed number.

Decimals word problems

Best for: 4th grade, 5th grade

47. Adding Decimals: You have 2.6 grams of yogurt in your bowl and you add another spoonful of 1.3 grams. How much yogurt do you have in total?

48. Subtracting Decimals: Gemma had 25.75 grams of frosting to make a cake. She decided to use only 15.5 grams of the frosting. How much frosting does Gemma have left?

49. Multiplying Decimals with Whole Numbers: Marshall walks a total of 0.9 kilometres to and from school each day. After 4 days, how many kilometres will he have walked?

50. Dividing Decimals by Whole Numbers: To make the Leaning Tower of Pisa from spaghetti, Mrs. Robinson bought 2.5 kilograms of spaghetti. Her students were able to make 10 leaning towers in total. How many kilograms of spaghetti does it take to make 1 leaning tower?

51. Mixing Addition and Subtraction of Decimals: Rocco has 1.5 litres of orange soda and 2.25 litres of grape soda in his fridge. Antonio has 1.15 litres of orange soda and 0.62 litres of grape soda. How much more soda does Rocco have than Angelo?

52. Mixing Multiplication and Division of Decimals: 4 days a week, Laura practices martial arts for 1.5 hours. Considering a week is 7 days, what is her average practice time per day each week?

Comparing and sequencing word problems

Four students are sitting together and discussing math questions

Best for: Kindergarten, 1st grade, 2nd grade

53. Comparing 1-Digit Integers: You have 3 apples and your friend has 5 apples. Who has more?

54. Comparing 2-Digit Integers: You have 50 candies and your friend has 75 candies. Who has more?

55. Comparing Different Variables: There are 5 basketballs on the playground. There are 7 footballs on the playground. Are there more basketballs or footballs?

56. Sequencing 1-Digit Integers: Erik has 0 stickers. Every day he gets 1 more sticker. How many days until he gets 3 stickers?

57. Skip-Counting by Odd Numbers: Natalie began at 5. She skip-counted by fives. Could she have said the number 20?

58. Skip-Counting by Even Numbers: Natasha began at 0. She skip-counted by eights. Could she have said the number 36?

59. Sequencing 2-Digit Numbers: Each month, Jeremy adds the same number of cards to his baseball card collection. In January, he had 36. 48 in February. 60 in March. How many baseball cards will Jeremy have in April?

Time word problems

66. Converting Hours into Minutes: Jeremy helped his mom for 1 hour. For how many minutes was he helping her?

69. Adding Time: If you wake up at 7:00 a.m. and it takes you 1 hour and 30 minutes to get ready and walk to school, at what time will you get to school?

70. Subtracting Time: If a train departs at 2:00 p.m. and arrives at 4:00 p.m., how long were passengers on the train for?

71. Finding Start and End Times: Rebecca left her dad’s store to go home at twenty to seven in the evening. Forty minutes later, she was home. What time was it when she arrived home?

Money word problems

Best for: 1st grade, 2nd grade, 3rd grade, 4th grade, 5th grade

60. Adding Money: Thomas and Matthew are saving up money to buy a video game together. Thomas has saved $30. Matthew has saved $35. How much money have they saved up together in total?

61. Subtracting Money: Thomas has $80 saved up. He uses his money to buy a video game. The video game costs $67. How much money does he have left?

62. Multiplying Money: Tim gets $5 for delivering the paper. How much money will he have after delivering the paper 3 times?

63. Dividing Money: Robert spent $184.59 to buy 3 hockey sticks. If each hockey stick was the same price, how much did 1 cost?

64. Adding Money with Decimals: You went to the store and bought gum for $1.25 and a sucker for $0.50. How much was your total?

65. Subtracting Money with Decimals: You went to the store with $5.50. You bought gum for $1.25, a chocolate bar for $1.15 and a sucker for $0.50. How much money do you have left?

67. Applying Proportional Relationships to Money: Jakob wants to invite 20 friends to his birthday, which will cost his parents $250. If he decides to invite 15 friends instead, how much money will it cost his parents? Assume the relationship is directly proportional.

68. Applying Percentages to Money: Retta put $100.00 in a bank account that gains 20% interest annually. How much interest will be accumulated in 1 year? And if she makes no withdrawals, how much money will be in the account after 1 year?

Physical measurement word problems

A girl is doing math practice

Best for: 1st grade, 2nd grade, 3rd grade, 4th grade

72. Comparing Measurements: Cassandra’s ruler is 22 centimetres long. April’s ruler is 30 centimetres long. How many centimetres longer is April’s ruler?

73. Contextualizing Measurements: Picture a school bus. Which unit of measurement would best describe the length of the bus? Centimetres, metres or kilometres?

74. Adding Measurements: Micha’s dad wants to try to save money on gas, so he has been tracking how much he uses. Last year, Micha’s dad used 100 litres of gas. This year, her dad used 90 litres of gas. How much gas did he use in total for the two years?

75. Subtracting Measurements: Micha’s dad wants to try to save money on gas, so he has been tracking how much he uses. Over the past two years, Micha’s dad used 200 litres of gas. This year, he used 100 litres of gas. How much gas did he use last year?

A tablet showing an example of Prodigy Math's battle gameplay.

76. Multiplying Volume and Mass: Kiera wants to make sure she has strong bones, so she drinks 2 litres of milk every week. After 3 weeks, how many litres of milk will Kiera drink?

77. Dividing Volume and Mass: Lillian is doing some gardening, so she bought 1 kilogram of soil. She wants to spread the soil evenly between her 2 plants. How much will each plant get?

78. Converting Mass: Inger goes to the grocery store and buys 3 squashes that each weigh 500 grams. How many kilograms of squash did Inger buy?

79. Converting Volume: Shad has a lemonade stand and sold 20 cups of lemonade. Each cup was 500 millilitres. How many litres did Shad sell in total?

80. Converting Length: Stacy and Milda are comparing their heights. Stacy is 1.5 meters tall. Milda is 10 centimetres taller than Stacy. What is Milda’s height in centimetres?

81. Understanding Distance and Direction: A bus leaves the school to take students on a field trip. The bus travels 10 kilometres south, 10 kilometres west, another 5 kilometres south and 15 kilometres north. To return to the school, in which direction does the bus have to travel? How many kilometres must it travel in that direction?

Ratios and percentages word problems

Best for: 4th grade, 5th grade, 6th grade

82. Finding a Missing Number: The ratio of Jenny’s trophies to Meredith’s trophies is 7:4. Jenny has 28 trophies. How many does Meredith have?

83. Finding Missing Numbers: The ratio of Jenny’s trophies to Meredith’s trophies is 7:4. The difference between the numbers is 12. What are the numbers?

84. Comparing Ratios: The school’s junior band has 10 saxophone players and 20 trumpet players. The school’s senior band has 18 saxophone players and 29 trumpet players. Which band has the higher ratio of trumpet to saxophone players?

85. Determining Percentages: Mary surveyed students in her school to find out what their favourite sports were. Out of 1,200 students, 455 said hockey was their favourite sport. What percentage of students said hockey was their favourite sport?

86. Determining Percent of Change: A decade ago, Oakville’s population was 67,624 people. Now, it is 190% larger. What is Oakville’s current population?

87. Determining Percents of Numbers: At the ice skate rental stand, 60% of 120 skates are for boys. If the rest of the skates are for girls, how many are there?

88. Calculating Averages: For 4 weeks, William volunteered as a helper for swimming classes. The first week, he volunteered for 8 hours. He volunteered for 12 hours in the second week, and another 12 hours in the third week. The fourth week, he volunteered for 9 hours. For how many hours did he volunteer per week, on average?

Probability and data relationships word problems

Two students are calculating on a whiteboard

Best for: 4th grade, 5th grade, 6th grade, 7th grade

89. Understanding the Premise of Probability: John wants to know his class’s favourite TV show, so he surveys all of the boys. Will the sample be representative or biased?

90. Understanding Tangible Probability: The faces on a fair number die are labelled 1, 2, 3, 4, 5 and 6. You roll the die 12 times. How many times should you expect to roll a 1?

91. Exploring Complementary Events: The numbers 1 to 50 are in a hat. If the probability of drawing an even number is 25/50, what is the probability of NOT drawing an even number? Express this probability as a fraction.

92. Exploring Experimental Probability: A pizza shop has recently sold 15 pizzas. 5 of those pizzas were pepperoni. Answering with a fraction, what is the experimental probability that he next pizza will be pepperoni?

93. Introducing Data Relationships: Maurita and Felice each take 4 tests. Here are the results of Maurita’s 4 tests: 4, 4, 4, 4. Here are the results for 3 of Felice’s 4 tests: 3, 3, 3. If Maurita’s mean for the 4 tests is 1 point higher than Felice’s, what’s the score of Felice’s 4th test?

94. Introducing Proportional Relationships: Store A is selling 7 pounds of bananas for $7.00. Store B is selling 3 pounds of bananas for $6.00. Which store has the better deal?

95. Writing Equations for Proportional Relationships: Lionel loves soccer, but has trouble motivating himself to practice. So, he incentivizes himself through video games. There is a proportional relationship between the amount of drills Lionel completes, in x , and for how many hours he plays video games, in y . When Lionel completes 10 drills, he plays video games for 30 minutes. Write the equation for the relationship between x and y .

Geometry word problems

Best for: 4th grade, 5th grade, 6th grade, 7th grade, 8th grade

96. Introducing Perimeter:  The theatre has 4 chairs in a row. There are 5 rows. Using rows as your unit of measurement, what is the perimeter?

97. Introducing Area: The theatre has 4 chairs in a row. There are 5 rows. How many chairs are there in total?

98. Introducing Volume: Aaron wants to know how much candy his container can hold. The container is 20 centimetres tall, 10 centimetres long and 10 centimetres wide. What is the container’s volume?

99. Understanding 2D Shapes: Kevin draws a shape with 4 equal sides. What shape did he draw?

100. Finding the Perimeter of 2D Shapes: Mitchell wrote his homework questions on a piece of square paper. Each side of the paper is 8 centimetres. What is the perimeter?

101. Determining the Area of 2D Shapes: A single trading card is 9 centimetres long by 6 centimetres wide. What is its area?

102. Understanding 3D Shapes: Martha draws a shape that has 6 square faces. What shape did she draw?

103. Determining the Surface Area of 3D Shapes: What is the surface area of a cube that has a width of 2cm, height of 2 cm and length of 2 cm?

104. Determining the Volume of 3D Shapes: Aaron’s candy container is 20 centimetres tall, 10 centimetres long and 10 centimetres wide. Bruce’s container is 25 centimetres tall, 9 centimetres long and 9 centimetres wide. Find the volume of each container. Based on volume, whose container can hold more candy?

105. Identifying Right-Angled Triangles: A triangle has the following side lengths: 3 cm, 4 cm and 5 cm. Is this triangle a right-angled triangle?

106. Identifying Equilateral Triangles: A triangle has the following side lengths: 4 cm, 4 cm and 4 cm. What kind of triangle is it?

107. Identifying Isosceles Triangles: A triangle has the following side lengths: 4 cm, 5 cm and 5 cm. What kind of triangle is it?

108. Identifying Scalene Triangles: A triangle has the following side lengths: 4 cm, 5 cm and 6 cm. What kind of triangle is it?

109. Finding the Perimeter of Triangles: Luigi built a tent in the shape of an equilateral triangle. The perimeter is 21 metres. What is the length of each of the tent’s sides?

110. Determining the Area of Triangles: What is the area of a triangle with a base of 2 units and a height of 3 units?

111. Applying Pythagorean Theorem: A right triangle has one non-hypotenuse side length of 3 inches and the hypotenuse measures 5 inches. What is the length of the other non-hypotenuse side?

112. Finding a Circle’s Diameter: Jasmin bought a new round backpack. Its area is 370 square centimetres. What is the round backpack’s diameter?

113. Finding a Circle's Area: Captain America’s circular shield has a diameter of 76.2 centimetres. What is the area of his shield?

114. Finding a Circle’s Radius: Skylar lives on a farm, where his dad keeps a circular corn maze. The corn maze has a diameter of 2 kilometres. What is the maze’s radius?

Variables word problems

A hand is calculating math problem on a blacboard

Best for: 6th grade, 7th grade, 8th grade

115. Identifying Independent and Dependent Variables: Victoria is baking muffins for her class. The number of muffins she makes is based on how many classmates she has. For this equation, m is the number of muffins and c is the number of classmates. Which variable is independent and which variable is dependent?

116. Writing Variable Expressions for Addition: Last soccer season, Trish scored g goals. Alexa scored 4 more goals than Trish. Write an expression that shows how many goals Alexa scored.

117. Writing Variable Expressions for Subtraction: Elizabeth eats a healthy, balanced breakfast b times a week. Madison sometimes skips breakfast. In total, Madison eats 3 fewer breakfasts a week than Elizabeth. Write an expression that shows how many times a week Madison eats breakfast.

118. Writing Variable Expressions for Multiplication: Last hockey season, Jack scored g goals. Patrik scored twice as many goals than Jack. Write an expression that shows how many goals Patrik scored.

119. Writing Variable Expressions for Division: Amanda has c chocolate bars. She wants to distribute the chocolate bars evenly among 3 friends. Write an expression that shows how many chocolate bars 1 of her friends will receive.

120. Solving Two-Variable Equations: This equation shows how the amount Lucas earns from his after-school job depends on how many hours he works: e = 12h . The variable h represents how many hours he works. The variable e represents how much money he earns. How much money will Lucas earn after working for 6 hours?

How to easily make your own math word problems & word problems worksheets

Two teachers are discussing math with a pen and a notebook

Armed with 120 examples to spark ideas, making your own math word problems can engage your students and ensure alignment with lessons. Do:

  • Link to Student Interests:  By framing your word problems with student interests, you’ll likely grab attention. For example, if most of your class loves American football, a measurement problem could involve the throwing distance of a famous quarterback.
  • Make Questions Topical:  Writing a word problem that reflects current events or issues can engage students by giving them a clear, tangible way to apply their knowledge.
  • Include Student Names:  Naming a question’s characters after your students is an easy way make subject matter relatable, helping them work through the problem.
  • Be Explicit:  Repeating keywords distills the question, helping students focus on the core problem.
  • Test Reading Comprehension:  Flowery word choice and long sentences can hide a question’s key elements. Instead, use concise phrasing and grade-level vocabulary.
  • Focus on Similar Interests:  Framing too many questions with related interests -- such as football and basketball -- can alienate or disengage some students.
  • Feature Red Herrings:  Including unnecessary information introduces another problem-solving element, overwhelming many elementary students.

A key to differentiated instruction , word problems that students can relate to and contextualize will capture interest more than generic and abstract ones.

Solver Title

Practice

Generating PDF...

  • Pre Algebra Order of Operations Factors & Primes Fractions Long Arithmetic Decimals Exponents & Radicals Ratios & Proportions Percent Modulo Number Line Mean, Median & Mode
  • Algebra Equations Inequalities System of Equations System of Inequalities Basic Operations Algebraic Properties Partial Fractions Polynomials Rational Expressions Sequences Power Sums Interval Notation Pi (Product) Notation Induction Logical Sets Word Problems
  • Pre Calculus Equations Inequalities Scientific Calculator Scientific Notation Arithmetics Complex Numbers Polar/Cartesian Simultaneous Equations System of Inequalities Polynomials Rationales Functions Arithmetic & Comp. Coordinate Geometry Plane Geometry Solid Geometry Conic Sections Trigonometry
  • Calculus Derivatives Derivative Applications Limits Integrals Integral Applications Integral Approximation Series ODE Multivariable Calculus Laplace Transform Taylor/Maclaurin Series Fourier Series Fourier Transform
  • Functions Line Equations Functions Arithmetic & Comp. Conic Sections Transformation
  • Linear Algebra Matrices Vectors
  • Trigonometry Identities Proving Identities Trig Equations Trig Inequalities Evaluate Functions Simplify
  • Statistics Mean Geometric Mean Quadratic Mean Average Median Mode Order Minimum Maximum Probability Mid-Range Range Standard Deviation Variance Lower Quartile Upper Quartile Interquartile Range Midhinge Standard Normal Distribution
  • Physics Mechanics
  • Chemistry Chemical Reactions Chemical Properties
  • Finance Simple Interest Compound Interest Present Value Future Value
  • Economics Point of Diminishing Return
  • Conversions Roman Numerals Radical to Exponent Exponent to Radical To Fraction To Decimal To Mixed Number To Improper Fraction Radians to Degrees Degrees to Radians Hexadecimal Scientific Notation Distance Weight Time Volume
  • Pre Algebra
  • One-Step Addition
  • One-Step Subtraction
  • One-Step Multiplication
  • One-Step Division
  • One-Step Decimals
  • Two-Step Integers
  • Two-Step Add/Subtract
  • Two-Step Multiply/Divide
  • Two-Step Fractions
  • Two-Step Decimals
  • Multi-Step Integers
  • Multi-Step with Parentheses
  • Multi-Step Rational
  • Multi-Step Fractions
  • Multi-Step Decimals
  • Solve by Factoring
  • Completing the Square
  • Quadratic Formula
  • Biquadratic
  • Logarithmic
  • Exponential
  • Rational Roots
  • Floor/Ceiling
  • Equation Given Roots
  • Newton Raphson
  • Substitution
  • Elimination
  • Cramer's Rule
  • Gaussian Elimination
  • System of Inequalities
  • Perfect Squares
  • Difference of Squares
  • Difference of Cubes
  • Sum of Cubes
  • Polynomials
  • Distributive Property
  • FOIL method
  • Perfect Cubes
  • Binomial Expansion
  • Negative Rule
  • Product Rule
  • Quotient Rule
  • Expand Power Rule
  • Fraction Exponent
  • Exponent Rules
  • Exponential Form
  • Logarithmic Form
  • Absolute Value
  • Rational Number
  • Powers of i
  • Partial Fractions
  • Is Polynomial
  • Leading Coefficient
  • Leading Term
  • Standard Form
  • Complete the Square
  • Synthetic Division
  • Linear Factors
  • Rationalize Denominator
  • Rationalize Numerator
  • Identify Type
  • Convergence
  • Interval Notation
  • Pi (Product) Notation
  • Boolean Algebra
  • Truth Table
  • Mutual Exclusive
  • Cardinality
  • Caretesian Product
  • Age Problems
  • Distance Problems
  • Cost Problems
  • Investment Problems
  • Number Problems
  • Percent Problems
  • Addition/Subtraction
  • Multiplication/Division
  • Dice Problems
  • Coin Problems
  • Card Problems
  • Pre Calculus
  • Linear Algebra
  • Trigonometry
  • Conversions

Click to reveal more operations

Most Used Actions

Number line.

  • \mathrm{Lauren's\:age\:is\:half\:of\:Joe's\:age.\:Emma\:is\:four\:years\:older\:than\:Joe.\:The\:sum\:of\:Lauren,\:Emma,\:and\:Joe's\:age\:is\:54.\:How\:old\:is\:Joe?}
  • \mathrm{Kira\:went\:for\:a\:drive\:in\:her\:new\:car.\:She\:drove\:for\:142.5\:miles\:at\:a\:speed\:of\:57\:mph.\:For\:how\:many\:hours\:did\:she\:drive?}
  • \mathrm{The\:sum\:of\:two\:numbers\:is\:249\:.\:Twice\:the\:larger\:number\:plus\:three\:times\:the\:smaller\:number\:is\:591\:.\:Find\:the\:numbers.}
  • \mathrm{If\:2\:tacos\:and\:3\:drinks\:cost\:12\:and\:3\:tacos\:and\:2\:drinks\:cost\:13\:how\:much\:does\:a\:taco\:cost?}
  • \mathrm{You\:deposit\:3000\:in\:an\:account\:earning\:2\%\:interest\:compounded\:monthly.\:How\:much\:will\:you\:have\:in\:the\:account\:in\:15\:years?}
  • How do you solve word problems?
  • To solve word problems start by reading the problem carefully and understanding what it's asking. Try underlining or highlighting key information, such as numbers and key words that indicate what operation is needed to perform. Translate the problem into mathematical expressions or equations, and use the information and equations generated to solve for the answer.
  • How do you identify word problems in math?
  • Word problems in math can be identified by the use of language that describes a situation or scenario. Word problems often use words and phrases which indicate that performing calculations is needed to find a solution. Additionally, word problems will often include specific information such as numbers, measurements, and units that needed to be used to solve the problem.
  • Is there a calculator that can solve word problems?
  • Symbolab is the best calculator for solving a wide range of word problems, including age problems, distance problems, cost problems, investments problems, number problems, and percent problems.
  • What is an age problem?
  • An age problem is a type of word problem in math that involves calculating the age of one or more people at a specific point in time. These problems often use phrases such as 'x years ago,' 'in y years,' or 'y years later,' which indicate that the problem is related to time and age.

word-problems-calculator

  • Middle School Math Solutions – Inequalities Calculator Next up in our Getting Started maths solutions series is help with another middle school algebra topic - solving...

Please add a message.

Message received. Thanks for the feedback.

Module 9: Multi-Step Linear Equations

Apply a problem-solving strategy to basic word problems, learning outcomes.

  • Practice mindfulness with your attitude about word problems
  • Apply a general problem-solving strategy to solve word problems

 Approach Word Problems with a Positive Attitude

The world is full of word problems. How much money do I need to fill the car with gas? How much should I tip the server at a restaurant? How many socks should I pack for vacation? How big a turkey do I need to buy for Thanksgiving dinner, and what time do I need to put it in the oven? If my sister and I buy our mother a present, how much will each of us pay?

Now that we can solve equations, we are ready to apply our new skills to word problems. Do you know anyone who has had negative experiences in the past with word problems? Have you ever had thoughts like the student in the cartoon below?

A cartoon image of a girl with a sad expression writing on a piece of paper is shown. There are 5 thought bubbles. They read,

Negative thoughts about word problems can be barriers to success.

When we feel we have no control, and continue repeating negative thoughts, we set up barriers to success. We need to calm our fears and change our negative feelings.

Start with a fresh slate and begin to think positive thoughts like the student in the cartoon below. Read the positive thoughts and say them out loud.

A cartoon image of a girl with a confident expression holding some books is shown. There are 4 thought bubbles. They read,

When it comes to word problems, a positive attitude is a big step toward success.

If we take control and believe we can be successful, we will be able to master word problems.

Think of something that you can do now but couldn’t do three years ago. Whether it’s driving a car, snowboarding, cooking a gourmet meal, or speaking a new language, you have been able to learn and master a new skill. Word problems are no different. Even if you have struggled with word problems in the past, you have acquired many new math skills that will help you succeed now!

Use a Problem-Solving Strategy for Word Problems

In earlier chapters, you translated word phrases into algebraic expressions, using some basic mathematical vocabulary and symbols. Since then you’ve increased your math vocabulary as you learned about more algebraic procedures, and you’ve had more practice translating from words into algebra.

You have also translated word sentences into algebraic equations and solved some word problems. The word problems applied math to everyday situations. You had to restate the situation in one sentence, assign a variable, and then write an equation to solve. This method works as long as the situation is familiar to you and the math is not too complicated.

Now we’ll develop a strategy you can use to solve any word problem. This strategy will help you become successful with word problems. We’ll demonstrate the strategy as we solve the following problem.

Pete bought a shirt on sale for $[latex]18[/latex], which is one-half the original price. What was the original price of the shirt?

Solution: Step 1. Read the problem. Make sure you understand all the words and ideas. You may need to read the problem two or more times. If there are words you don’t understand, look them up in a dictionary or on the Internet.

  • In this problem, do you understand what is being discussed? Do you understand every word?

Step 2. Identify what you are looking for. It’s hard to find something if you are not sure what it is! Read the problem again and look for words that tell you what you are looking for!

  • In this problem, the words “what was the original price of the shirt” tell you what you are looking for: the original price of the shirt.

Step 3. Name what you are looking for. Choose a variable to represent that quantity. You can use any letter for the variable, but it may help to choose one that helps you remember what it represents.

  • Let [latex]p=[/latex] the original price of the shirt

Step 4. Translate into an equation. It may help to first restate the problem in one sentence, with all the important information. Then translate the sentence into an equation.

The top line reads:

Step 6. Check the answer in the problem and make sure it makes sense.

  • We found that [latex]p=36[/latex], which means the original price was [latex]\text{\$36}[/latex]. Does [latex]\text{\$36}[/latex] make sense in the problem? Yes, because [latex]18[/latex] is one-half of [latex]36[/latex], and the shirt was on sale at half the original price.

Step 7. Answer the question with a complete sentence.

  • The problem asked “What was the original price of the shirt?” The answer to the question is: “The original price of the shirt was [latex]\text{\$36}[/latex].”

If this were a homework exercise, our work might look like this:

The top reads,

We list the steps we took to solve the previous example.

Problem-Solving Strategy

  • Read the word problem. Make sure you understand all the words and ideas. You may need to read the problem two or more times. If there are words you don’t understand, look them up in a dictionary or on the internet.
  • Identify what you are looking for.
  • Name what you are looking for. Choose a variable to represent that quantity.
  • Translate into an equation. It may be helpful to first restate the problem in one sentence before translating.
  • Solve the equation using good algebra techniques.
  • Check the answer in the problem. Make sure it makes sense.
  • Answer the question with a complete sentence.

For a review of how to translate algebraic statements into words, watch the following video.

Let’s use this approach with another example.

Yash brought apples and bananas to a picnic. The number of apples was three more than twice the number of bananas. Yash brought [latex]11[/latex] apples to the picnic. How many bananas did he bring?

In the next example, we will apply our Problem-Solving Strategy to applications of percent.

Nga’s car insurance premium increased by [latex]\text{\$60}[/latex], which was [latex]\text{8%}[/latex] of the original cost. What was the original cost of the premium?

  • Write Algebraic Expressions from Statements: Form ax+b and a(x+b). Authored by : James Sousa (Mathispower4u.com) for Lumen Learning. Located at : https://youtu.be/Hub7ku7UHT4 . License : CC BY: Attribution
  • Question ID 142694, 142722, 142735, 142761. Authored by : Lumen Learning. License : CC BY: Attribution . License Terms : IMathAS Community License, CC-BY + GPL
  • Prealgebra. Provided by : OpenStax. License : CC BY: Attribution . License Terms : Download for free at http://cnx.org/contents/[email protected]

Footer Logo Lumen Waymaker

Math Word Problems

Welcome to the math word problems worksheets page at Math-Drills.com! On this page, you will find Math word and story problems worksheets with single- and multi-step solutions on a variety of math topics including addition, multiplication, subtraction, division and other math topics. It is usually a good idea to ensure students already have a strategy or two in place to complete the math operations involved in a particular question. For example, students may need a way to figure out what 7 × 8 is or have previously memorized the answer before you give them a word problem that involves finding the answer to 7 × 8.

There are a number of strategies used in solving math word problems; if you don't have a favorite, try the Math-Drills.com problem-solving strategy:

  • Question : Understand what the question is asking. What operation or operations do you need to use to solve this question? Ask for help to understand the question if you can't do it on your own.
  • Estimate : Use an estimation strategy, so you can check your answer for reasonableness in the evaluate step. Try underestimating and overestimating, so you know what range the answer is supposed to be in. Be flexible in rounding numbers if it will make your estimate easier.
  • Strategize : Choose a strategy to solve the problem. Will you use mental math, manipulatives, or pencil and paper? Use a strategy that works for you. Save the calculator until the evaluate stage.
  • Calculate : Use your strategy to solve the problem.
  • Evaluate : Compare your answer to your estimate. If you under and overestimated, is the answer in the correct range. If you rounded up or down, does the answer make sense (e.g. is it a little less or a little more than the estimate). Also check with a calculator.

Most Popular Math Word Problems this Week

Easy Multi-Step Word Problems

Arithmetic Word Problems

problem solving involving word problems

  • Addition Word Problems One-Step Addition Word Problems Using Single-Digit Numbers One-Step Addition Word Problems Using Two-Digit Numbers
  • Subtraction Word Problems Subtraction Facts Word Problems With Differences from 5 to 12
  • Multiplication Word Problems One-Step Multiplication Word Problems up to 10 × 10
  • Division Word Problems Division Facts Word Problems with Quotients from 5 to 12
  • Multi-Step Word Problems Easy Multi-Step Word Problems

Copyright © 2005-2024 Math-Drills.com You may use the math worksheets on this website according to our Terms of Use to help students learn math.

Library homepage

  • school Campus Bookshelves
  • menu_book Bookshelves
  • perm_media Learning Objects
  • login Login
  • how_to_reg Request Instructor Account
  • hub Instructor Commons
  • Download Page (PDF)
  • Download Full Book (PDF)
  • Periodic Table
  • Physics Constants
  • Scientific Calculator
  • Reference & Cite
  • Tools expand_more
  • Readability

selected template will load here

This action is not available.

Mathematics LibreTexts

1.20: Word Problems for Linear Equations

  • Last updated
  • Save as PDF
  • Page ID 45640

Word problems are important applications of linear equations. We start with examples of translating an English sentence or phrase into an algebraic expression.

Example 18.1

Translate the phrase into an algebraic expression:

a) Twice a variable is added to 4

Solution: We call the variable \(x .\) Twice the variable is \(2 x .\) Adding \(2 x\) to 4 gives:

\[4 + 2x\nonumber\]

b) Three times a number is subtracted from 7.

Solution: Three times a number is \(3 x .\) We need to subtract \(3 x\) from 7. This means:\

\[7-3 x\nonumber\]

c) 8 less than a number.

Solution: The number is denoted by \(x .8\) less than \(x\) mean, that we need to subtract 8 from it. We get:

\[x-8\nonumber\]

For example, 8 less than 10 is \(10-8=2\).

d) Subtract \(5 p^{2}-7 p+2\) from \(3 p^{2}+4 p\) and simplify.

Solution: We need to calculate \(3 p^{2}+4 p\) minus \(5 p^{2}-7 p+2:\)

\[\left(3 p^{2}+4 p\right)-\left(5 p^{2}-7 p+2\right)\nonumber\]

Simplifying this expression gives:

\[\left(3 p^{2}+4 p\right)-\left(5 p^{2}-7 p+2\right)=3 p^{2}+4 p-5 p^{2}+7 p-2 =-2 p^{2}+11 p-2\nonumber\]

e) The amount of money given by \(x\) dimes and \(y\) quarters.

Solution: Each dime is worth 10 cents, so that this gives a total of \(10 x\) cents. Each quarter is worth 25 cents, so that this gives a total of \(25 y\) cents. Adding the two amounts gives a total of

\[10 x+25 y \text{ cents or } .10x + .25y \text{ dollars}\nonumber\]

Now we deal with word problems that directly describe an equation involving one variable, which we can then solve.

Example 18.2

Solve the following word problems:

a) Five times an unknown number is equal to 60. Find the number.

Solution: We translate the problem to algebra:

\[5x = 60\nonumber\]

We solve this for \(x\) :

\[x=\frac{60}{5}=12\nonumber\]

b) If 5 is subtracted from twice an unknown number, the difference is \(13 .\) Find the number.

Solution: Translating the problem into an algebraic equation gives:

\[2x − 5 = 13\nonumber\]

We solve this for \(x\). First, add 5 to both sides.

\[2x = 13 + 5, \text{ so that } 2x = 18\nonumber\]

Dividing by 2 gives \(x=\frac{18}{2}=9\).

c) A number subtracted from 9 is equal to 2 times the number. Find the number.

Solution: We translate the problem to algebra.

\[9 − x = 2x\nonumber\]

We solve this as follows. First, add \(x\) :

\[9 = 2x + x \text{ so that } 9 = 3x\nonumber\]

Then the answer is \(x=\frac{9}{3}=3\)

d) Multiply an unknown number by five is equal to adding twelve to the unknown number. Find the number.

Solution: We have the equation:

\[5x = x + 12.\nonumber\]

Subtracting \(x\) gives

\[4x = 12.\nonumber\]

Dividing both sides by 4 gives the answer: \(x=3\).

e) Adding nine to a number gives the same result as subtracting seven from three times the number. Find the number.

Solution: Adding 9 to a number is written as \(x+9,\) while subtracting 7 from three times the number is written as \(3 x-7\). We therefore get the equation:

\[x + 9 = 3x − 7.\nonumber\]

We solve for \(x\) by adding 7 on both sides of the equation:

\[x + 16 = 3x.\nonumber\]

Then we subtract \(x:\)

\[16 = 2x.\nonumber\]

After dividing by \(2,\) we obtain the answer \(x=8\)

The following word problems consider real world applications. They require to model a given situation in the form of an equation.

Example 18.3

a) Due to inflation, the price of a loaf of bread has increased by \(5 \%\). How much does the loaf of bread cost now, when its price was \(\$ 2.40\) last year?

Solution: We calculate the price increase as \(5 \% \cdot \$ 2.40 .\) We have

\[5 \% \cdot 2.40=0.05 \cdot 2.40=0.1200=0.12\nonumber\]

We must add the price increase to the old price.

\[2.40+0.12=2.52\nonumber\]

The new price is therefore \(\$ 2.52\).

b) To complete a job, three workers get paid at a rate of \(\$ 12\) per hour. If the total pay for the job was \(\$ 180,\) then how many hours did the three workers spend on the job?

Solution: We denote the number of hours by \(x\). Then the total price is calculated as the price per hour \((\$ 12)\) times the number of workers times the number of hours \((3) .\) We obtain the equation

\[12 \cdot 3 \cdot x=180\nonumber\]

Simplifying this yields

\[36 x=180\nonumber\]

Dividing by 36 gives

\[x=\frac{180}{36}=5\nonumber\]

Therefore, the three workers needed 5 hours for the job.

c) A farmer cuts a 300 foot fence into two pieces of different sizes. The longer piece should be four times as long as the shorter piece. How long are the two pieces?

\[x+4 x=300\nonumber\]

Combining the like terms on the left, we get

\[5 x=300\nonumber\]

Dividing by 5, we obtain that

\[x=\frac{300}{5}=60\nonumber\]

Therefore, the shorter piece has a length of 60 feet, while the longer piece has four times this length, that is \(4 \times 60\) feet \(=240\) feet.

d) If 4 blocks weigh 28 ounces, how many blocks weigh 70 ounces?

Solution: We denote the weight of a block by \(x .\) If 4 blocks weigh \(28,\) then a block weighs \(x=\frac{28}{4}=7\)

How many blocks weigh \(70 ?\) Well, we only need to find \(\frac{70}{7}=10 .\) So, the answer is \(10 .\)

Note You can solve this problem by setting up and solving the fractional equation \(\frac{28}{4}=\frac{70}{x}\). Solving such equations is addressed in chapter 24.

e) If a rectangle has a length that is three more than twice the width and the perimeter is 20 in, what are the dimensions of the rectangle?

Solution: We denote the width by \(x\). Then the length is \(2 x+3\). The perimeter is 20 in on one hand and \(2(\)length\()+2(\)width\()\) on the other. So we have

\[20=2 x+2(2 x+3)\nonumber\]

Distributing and collecting like terms give

\[20=6 x+6\nonumber\]

Subtracting 6 from both sides of the equation and then dividing both sides of the resulting equation by 6 gives:

\[20-6=6 x \Longrightarrow 14=6 x \Longrightarrow x=\frac{14}{6} \text { in }=\frac{7}{3} \text { in }=2 \frac{1}{3} \text { in. }\nonumber\]

f) If a circle has circumference 4in, what is its radius?

Solution: We know that \(C=2 \pi r\) where \(C\) is the circumference and \(r\) is the radius. So in this case

\[4=2 \pi r\nonumber\]

Dividing both sides by \(2 \pi\) gives

\[r=\frac{4}{2 \pi}=\frac{2}{\pi} \text { in } \approx 0.63 \mathrm{in}\nonumber\]

g) The perimeter of an equilateral triangle is 60 meters. How long is each side?

Solution: Let \(x\) equal the side of the triangle. Then the perimeter is, on the one hand, \(60,\) and on other hand \(3 x .\) So \(3 x=60\) and dividing both sides of the equation by 3 gives \(x=20\) meters.

h) If a gardener has \(\$ 600\) to spend on a fence which costs \(\$ 10\) per linear foot and the area to be fenced in is rectangular and should be twice as long as it is wide, what are the dimensions of the largest fenced in area?

Solution: The perimeter of a rectangle is \(P=2 L+2 W\). Let \(x\) be the width of the rectangle. Then the length is \(2 x .\) The perimeter is \(P=2(2 x)+2 x=6 x\). The largest perimeter is \(\$ 600 /(\$ 10 / f t)=60\) ft. So \(60=6 x\) and dividing both sides by 6 gives \(x=60 / 6=10\). So the dimensions are 10 feet by 20 feet.

i) A trapezoid has an area of 20.2 square inches with one base measuring 3.2 in and the height of 4 in. Find the length of the other base.

Solution: Let \(b\) be the length of the unknown base. The area of the trapezoid is on the one hand 20.2 square inches. On the other hand it is \(\frac{1}{2}(3.2+b) \cdot 4=\) \(6.4+2 b .\) So

\[20.2=6.4+2 b\nonumber\]

Multiplying both sides by 10 gives

\[202=64+20 b\nonumber\]

Subtracting 64 from both sides gives

\[b=\frac{138}{20}=\frac{69}{10}=6.9 \text { in }\nonumber\]

and dividing by 20 gives

Exit Problem

Write an equation and solve: A car uses 12 gallons of gas to travel 100 miles. How many gallons would be needed to travel 450 miles?

If you're seeing this message, it means we're having trouble loading external resources on our website.

If you're behind a web filter, please make sure that the domains *.kastatic.org and *.kasandbox.org are unblocked.

To log in and use all the features of Khan Academy, please enable JavaScript in your browser.

Praxis Core Math

Course: praxis core math   >   unit 1.

  • Algebraic properties | Lesson
  • Algebraic properties | Worked example
  • Solution procedures | Lesson
  • Solution procedures | Worked example
  • Equivalent expressions | Lesson
  • Equivalent expressions | Worked example
  • Creating expressions and equations | Lesson
  • Creating expressions and equations | Worked example

Algebraic word problems | Lesson

  • Algebraic word problems | Worked example
  • Linear equations | Lesson
  • Linear equations | Worked example
  • Quadratic equations | Lesson
  • Quadratic equations | Worked example

What are algebraic word problems?

What skills are needed.

  • Translating sentences to equations
  • Solving linear equations with one variable
  • Evaluating algebraic expressions
  • Solving problems using Venn diagrams

How do we solve algebraic word problems?

  • Define a variable.
  • Write an equation using the variable.
  • Solve the equation.
  • If the variable is not the answer to the word problem, use the variable to calculate the answer.

What's a Venn diagram?

  • 7 + 10 − 13 = 4 ‍   brought both food and drinks.
  • 7 − 4 = 3 ‍   brought only food.
  • 10 − 4 = 6 ‍   brought only drinks.
  • Your answer should be
  • an integer, like 6 ‍  
  • a simplified proper fraction, like 3 / 5 ‍  
  • a simplified improper fraction, like 7 / 4 ‍  
  • a mixed number, like 1   3 / 4 ‍  
  • an exact decimal, like 0.75 ‍  
  • a multiple of pi, like 12   pi ‍   or 2 / 3   pi ‍  
  • (Choice A)   $ 4 ‍   A $ 4 ‍  
  • (Choice B)   $ 5 ‍   B $ 5 ‍  
  • (Choice C)   $ 9 ‍   C $ 9 ‍  
  • (Choice D)   $ 14 ‍   D $ 14 ‍  
  • (Choice E)   $ 20 ‍   E $ 20 ‍  
  • (Choice A)   10 ‍   A 10 ‍  
  • (Choice B)   12 ‍   B 12 ‍  
  • (Choice C)   24 ‍   C 24 ‍  
  • (Choice D)   30 ‍   D 30 ‍  
  • (Choice E)   32 ‍   E 32 ‍  
  • (Choice A)   4 ‍   A 4 ‍  
  • (Choice B)   10 ‍   B 10 ‍  
  • (Choice C)   14 ‍   C 14 ‍  
  • (Choice D)   18 ‍   D 18 ‍  
  • (Choice E)   22 ‍   E 22 ‍  

Things to remember

Want to join the conversation.

  • Upvote Button navigates to signup page
  • Downvote Button navigates to signup page
  • Flag Button navigates to signup page

Radical Equation Word Problems - Examples & Practice - Expii

Radical equation word problems - examples & practice, explanations (3).

The key to solving any word problem (whether it contains a radical or not) is to translate the problem from words into math .

That's the biggest step in word problems. Once you've translated the information into numbers, you solve the equation the same way as always.

Let's look at an example to see how this approach works when radicals are involved.

Hiroki is calculating the speed of a tsunami. The formula is speed=√9.8d where d is ocean depth in meters. In this case, the ocean was 1500 meters deep. How fast was the wave (in m/s2)?

So, in this problem (as with many problems involving radicals), the formula to use has already been given to us. We are interested in the speed of the wave, and we've been given the depth (d). So, let's start by plugging that value in : speed=√9.8(1500) Let's solve this problem.

How fast was the wave (m/s2)?

Related Lessons

Example walk through: word problem involving radicals.

The key to word problems is in the translation. Let's walk through the problem below and solve it.

Lori recently purchased a square painting with an area of 780 cm2. How much wood does Lori need to make the frame? Don't worry about the width and height of the wood, just find the length.

First, find out the essential information in the problem.

A square painting has area 780cm2, what is its side length?

What is the relationship between side length and area of a square? Area=side length×side length

Denote side length as x for simplicity, plug in the numbers we have: 780cm2=x×x Solve for x: 780=x2x=√78x≈27.93

Lastly, note that the question is asking how much wood Lori needs to build the frame. A square has 4 sides so we have to multiply 27.93cm by 4.

In total, Lori needs at least 27.93cm×4=111.72cm of wood.

Let's look at another problem!

problem solving involving word problems

Image source: By Caroline Kulczycky

problem solving involving word problems

(Video) Problem Solving with Radical Expressions

by Deanna Green

problem solving involving word problems

This video by Deanna Green works through word problems with radicals.

These problems usually focus around the Pythagorean Theorem . A helpful trick always is to draw a simple picture of what's going on. Then fill in lengths or angles as the problem states.

Let's work through one of the examples she looks at.

A kite is secured to a rope that is tied to the ground. A breeze blows the kite so that the rope is taught while the kite is directly above a flagpole that is 30 ft from where the rope is staked down. Find the altitude of the kite if the rope is 110 ft long.

The first step is to draw a diagram.

A diagram of the problem above.

Image source: by Alex Federspiel

The Pythagorean Theorem states:

The length of the hypotenuse (diagonal line) is equal to the square root of the square of the sides added.

As an equation:

where c is the hypotenuse and a and b are the other sides.

Start by plugging in all the information we know.

110=√302+x2

You cannot split up radicals with a sum underneath into two separate radicals. So, to solve this, we want to take the square of both sides.

(110)2=(√302+x2)212100=302+x212100−302=x212100−900=x211200=x2√11200=√x2105.83≈x

You may need to review the lesson on getting rid of the x2 in with this lesson .

Word Problems on Linear Equations

Worked-out word problems on linear equations with solutions explained step-by-step in different types of examples.

There are several problems which involve relations among known and unknown numbers and can be put in the form of equations. The equations are generally stated in words and it is for this reason we refer to these problems as word problems. With the help of equations in one variable, we have already practiced equations to solve some real life problems.

Steps involved in solving a linear equation word problem: ● Read the problem carefully and note what is given and what is required and what is given. ● Denote the unknown by the variables as x, y, ……. ● Translate the problem to the language of mathematics or mathematical statements. ● Form the linear equation in one variable using the conditions given in the problems. ● Solve the equation for the unknown. ● Verify to be sure whether the answer satisfies the conditions of the problem.

Step-by-step application of linear equations to solve practical word problems:

1. The sum of two numbers is 25. One of the numbers exceeds the other by 9. Find the numbers. 

Solution: Then the other number = x + 9 Let the number be x.  Sum of two numbers = 25 According to question, x + x + 9 = 25 ⇒ 2x + 9 = 25 ⇒ 2x = 25 - 9 (transposing 9 to the R.H.S changes to -9)  ⇒ 2x = 16 ⇒ 2x/2 = 16/2 (divide by 2 on both the sides)  ⇒ x = 8 Therefore, x + 9 = 8 + 9 = 17 Therefore, the two numbers are 8 and 17.

2.The difference between the two numbers is 48. The ratio of the two numbers is 7:3. What are the two numbers?  Solution:   Let the common ratio be x.  Let the common ratio be x.  Their difference = 48 According to the question,  7x - 3x = 48  ⇒ 4x = 48  ⇒ x = 48/4  ⇒ x = 12 Therefore, 7x = 7 × 12 = 84           3x = 3 × 12 = 36  Therefore, the two numbers are 84 and 36.

3. The length of a rectangle is twice its breadth. If the perimeter is 72 metre, find the length and breadth of the rectangle.  Solution: Let the breadth of the rectangle be x,  Then the length of the rectangle = 2x Perimeter of the rectangle = 72 Therefore, according to the question 2(x + 2x) = 72 ⇒ 2 × 3x = 72 ⇒ 6x = 72  ⇒ x = 72/6 ⇒ x = 12 We know, length of the rectangle = 2x                       = 2 × 12 = 24 Therefore, length of the rectangle is 24 m and breadth of the rectangle is 12 m.

4. Aaron is 5 years younger than Ron. Four years later, Ron will be twice as old as Aaron. Find their present ages. 

Solution: Let Ron’s present age be x.  Then Aaron’s present age = x - 5 After 4 years Ron’s age = x + 4, Aaron’s age x - 5 + 4.  According to the question;  Ron will be twice as old as Aaron.  Therefore, x + 4 = 2(x - 5 + 4)  ⇒ x + 4 = 2(x - 1)  ⇒ x + 4 = 2x - 2 ⇒ x + 4 = 2x - 2 ⇒ x - 2x = -2 - 4 ⇒ -x = -6 ⇒ x = 6 Therefore, Aaron’s present age = x - 5 = 6 - 5 = 1 Therefore, present age of Ron = 6 years and present age of Aaron = 1 year.

5. A number is divided into two parts, such that one part is 10 more than the other. If the two parts are in the ratio 5 : 3, find the number and the two parts.  Solution: Let one part of the number be x Then the other part of the number = x + 10 The ratio of the two numbers is 5 : 3 Therefore, (x + 10)/x = 5/3 ⇒ 3(x + 10) = 5x  ⇒ 3x + 30 = 5x ⇒ 30 = 5x - 3x ⇒ 30 = 2x  ⇒ x = 30/2  ⇒ x = 15 Therefore, x + 10 = 15 + 10 = 25 Therefore, the number = 25 + 15 = 40  The two parts are 15 and 25. 

More solved examples with detailed explanation on the word problems on linear equations.

6. Robert’s father is 4 times as old as Robert. After 5 years, father will be three times as old as Robert. Find their present ages.  Solution: Let Robert’s age be x years.  Then Robert’s father’s age = 4x After 5 years, Robert’s age = x + 5 Father’s age = 4x + 5 According to the question,  4x + 5 = 3(x + 5)  ⇒ 4x + 5 = 3x + 15  ⇒ 4x - 3x = 15 - 5  ⇒ x = 10 ⇒ 4x = 4 × 10 = 40  Robert’s present age is 10 years and that of his father’s age = 40 years.  

7. The sum of two consecutive multiples of 5 is 55. Find these multiples.  Solution: Let the first multiple of 5 be x.  Then the other multiple of 5 will be x + 5 and their sum = 55 Therefore, x + x + 5 = 55 ⇒ 2x + 5 = 55 ⇒ 2x = 55 - 5 ⇒ 2x = 50 ⇒ x = 50/2  ⇒ x = 25  Therefore, the multiples of 5, i.e., x + 5 = 25 + 5 = 30 Therefore, the two consecutive multiples of 5 whose sum is 55 are 25 and 30.  

8. The difference in the measures of two complementary angles is 12°. Find the measure of the angles.  Solution: Let the angle be x.  Complement of x = 90 - x Given their difference = 12° Therefore, (90 - x) - x = 12° ⇒ 90 - 2x = 12 ⇒ -2x = 12 - 90 ⇒ -2x = -78 ⇒ 2x/2 = 78/2 ⇒ x = 39 Therefore, 90 - x = 90 - 39 = 51  Therefore, the two complementary angles are 39° and 51°

9. The cost of two tables and three chairs is $705. If the table costs $40 more than the chair, find the cost of the table and the chair.  Solution: The table cost $ 40 more than the chair.  Let us assume the cost of the chair to be x.  Then the cost of the table = $ 40 + x The cost of 3 chairs = 3 × x = 3x and the cost of 2 tables 2(40 + x)  Total cost of 2 tables and 3 chairs = $705 Therefore, 2(40 + x) + 3x = 705 80 + 2x + 3x = 705 80 + 5x = 705 5x = 705 - 80 5x = 625/5 x = 125 and 40 + x = 40 + 125 = 165 Therefore, the cost of each chair is $125 and that of each table is $165. 

10. If 3/5 ᵗʰ of a number is 4 more than 1/2 the number, then what is the number?  Solution: Let the number be x, then 3/5 ᵗʰ of the number = 3x/5 Also, 1/2 of the number = x/2  According to the question,  3/5 ᵗʰ of the number is 4 more than 1/2 of the number.  ⇒ 3x/5 - x/2 = 4 ⇒ (6x - 5x)/10 = 4 ⇒ x/10 = 4 ⇒ x = 40 The required number is 40.  

Try to follow the methods of solving word problems on linear equations and then observe the detailed instruction on the application of equations to solve the problems.

●   Equations

What is an Equation?

What is a Linear Equation?

How to Solve Linear Equations?

Solving Linear Equations

Problems on Linear Equations in One Variable

Word Problems on Linear Equations in One Variable

Practice Test on Linear Equations

Practice Test on Word Problems on Linear Equations

●   Equations - Worksheets

Worksheet on Linear Equations

Worksheet on Word Problems on Linear Equation

7th Grade Math Problems   8th Grade Math Practice   From Word Problems on Linear Equations to HOME PAGE

New! Comments

Didn't find what you were looking for? Or want to know more information about Math Only Math . Use this Google Search to find what you need.

  • Preschool Activities
  • Kindergarten Math
  • 1st Grade Math
  • 2nd Grade Math
  • 3rd Grade Math
  • 4th Grade Math
  • 5th Grade Math
  • 6th Grade Math
  • 7th Grade Math
  • 8th Grade Math
  • 9th Grade Math
  • 10th Grade Math
  • 11 & 12 Grade Math
  • Concepts of Sets
  • Probability
  • Boolean Algebra
  • Math Coloring Pages
  • Multiplication Table
  • Cool Maths Games
  • Math Flash Cards
  • Online Math Quiz
  • Math Puzzles
  • Binary System
  • Math Dictionary
  • Conversion Chart
  • Homework Sheets
  • Math Problem Ans
  • Free Math Answers
  • Printable Math Sheet
  • Funny Math Answers
  • Employment Test
  • Math Patterns
  • Link Partners
  • Privacy Policy

XML RSS

Recent Articles

RSS

4th Grade Multiplication Worksheet | Math Multiplication Worksheets

Mar 31, 24 05:13 PM

Estimate the Product

Word Problems on Multiplication |Multiplication Word Problem Worksheet

Mar 31, 24 11:57 AM

Multiplication Word Problem

Multiplication by Ten, Hundred and Thousand |Multiply by 10, 100 &1000

Mar 31, 24 10:09 AM

Rules for the Multiplication by 10, 100 and 1000.

Properties of Multiplication | Multiplicative Identity | Whole Numbers

Mar 29, 24 05:08 PM

Properties of Multiplication of Whole Numbers

Multiplication of a Number by a 3-Digit Number |3-Digit Multiplication

Mar 28, 24 06:33 PM

Multiplying by 3-Digit Number

© and ™ math-only-math.com. All Rights Reserved. 2010 - 2024.

HIGH SCHOOL

  • ACT Tutoring
  • SAT Tutoring
  • PSAT Tutoring
  • ASPIRE Tutoring
  • SHSAT Tutoring
  • STAAR Tutoring

GRADUATE SCHOOL

  • MCAT Tutoring
  • GRE Tutoring
  • LSAT Tutoring
  • GMAT Tutoring
  • AIMS Tutoring
  • HSPT Tutoring
  • ISAT Tutoring
  • SSAT Tutoring

Search 50+ Tests

Loading Page

math tutoring

  • Elementary Math
  • Pre-Calculus
  • Trigonometry

science tutoring

Foreign languages.

  • Mandarin Chinese

elementary tutoring

  • Computer Science

Search 350+ Subjects

  • Video Overview
  • Tutor Selection Process
  • Online Tutoring
  • Mobile Tutoring
  • Instant Tutoring
  • How We Operate
  • Our Guarantee
  • Impact of Tutoring
  • Reviews & Testimonials
  • Media Coverage
  • About Varsity Tutors

SAT II Math I : Solving Functions from Word Problems

Study concepts, example questions & explanations for sat ii math i, all sat ii math i resources, example questions, example question #1 : solving functions from word problems.

At Joe's pizzeria a pizza costs $5 with the first topping, and then an additional 75 cents for each additional topping.

problem solving involving word problems

Notice that the question describes a linear equation because there is a constant rate of change (the cost per topping). This means we can use slope intercept form to describe the scenario. 

Recall that slope intercept form is

problem solving involving word problems

Putting all these steps together we get:

problem solving involving word problems

Report an issue with this question

If you've found an issue with this question, please let us know. With the help of the community we can continue to improve our educational resources.

DMCA Complaint

If you believe that content available by means of the Website (as defined in our Terms of Service) infringes one or more of your copyrights, please notify us by providing a written notice (“Infringement Notice”) containing the information described below to the designated agent listed below. If Varsity Tutors takes action in response to an Infringement Notice, it will make a good faith attempt to contact the party that made such content available by means of the most recent email address, if any, provided by such party to Varsity Tutors.

Your Infringement Notice may be forwarded to the party that made the content available or to third parties such as ChillingEffects.org.

Please be advised that you will be liable for damages (including costs and attorneys’ fees) if you materially misrepresent that a product or activity is infringing your copyrights. Thus, if you are not sure content located on or linked-to by the Website infringes your copyright, you should consider first contacting an attorney.

Please follow these steps to file a notice:

You must include the following:

A physical or electronic signature of the copyright owner or a person authorized to act on their behalf; An identification of the copyright claimed to have been infringed; A description of the nature and exact location of the content that you claim to infringe your copyright, in \ sufficient detail to permit Varsity Tutors to find and positively identify that content; for example we require a link to the specific question (not just the name of the question) that contains the content and a description of which specific portion of the question – an image, a link, the text, etc – your complaint refers to; Your name, address, telephone number and email address; and A statement by you: (a) that you believe in good faith that the use of the content that you claim to infringe your copyright is not authorized by law, or by the copyright owner or such owner’s agent; (b) that all of the information contained in your Infringement Notice is accurate, and (c) under penalty of perjury, that you are either the copyright owner or a person authorized to act on their behalf.

Send your complaint to our designated agent at:

Charles Cohn Varsity Tutors LLC 101 S. Hanley Rd, Suite 300 St. Louis, MO 63105

Or fill out the form below:

Contact Information

Complaint details.

Learning Tools by Varsity Tutors

20 Elapsed Time Word Problems

Classen Rafael / EyeEm / Getty Images

  • Classroom Organization
  • Reading Strategies
  • Becoming A Teacher
  • Assessments & Tests
  • Secondary Education
  • Special Education
  • Homeschooling

problem solving involving word problems

Elapsed time is the amount of time that passes between the beginning and the end of an event. The concept of elapsed time fits nicely in the elementary school curriculum. Beginning in third grade, students should be able to tell and write time to the nearest minute and solve word problems involving addition and subtraction of time. Reinforce these essential skills with the following elapsed time word problems and games.

Elapsed Time Word Problems

These quick and easy elapsed time word problems are perfect for parents and teachers who want to help students practice elapsed time to the nearest minute with simple mental math problems. Answers are listed below.

  • Sam and his mom arrive at the doctor’s office at 2:30 p.m. They see the doctor at 3:10 p.m. How long was their wait?
  • Dad says dinner will be ready in 35 minutes. It’s 5:30 p.m. now. What time will dinner be ready?
  • Becky is meeting her friend at the library at 12:45 p.m. It takes her 25 minutes to get to the library. What time will she need to leave her house to arrive on time?
  • Ethan’s birthday party started at 4:30 p.m. The last guest left at 6:32 p.m. How long did Ethan’s party last?
  • Kayla put cupcakes in the oven at 3:41 p.m. The directions say that the cupcakes need to bake for 38 minutes. What time will Kayla need to take them out of the oven?
  • Dakota arrived at school at 7:59 a.m. He left at 2:33 p.m. How long was Dakota at school?
  • Dylan started working on homework at 5:45 p.m. It took him 1 hour and 57 minutes to complete it. What time did Dylan complete his homework?
  • Dad arrives home at 4:50 p.m. He left work 40 minutes ago. What time did Dad get off work? 
  • Jessica’s family is traveling from Atlanta, Georgia to New York by plane. Their flight leaves at 11:15 a.m. and should take 2 hours and 15 minutes. What time will their plane arrive in New York?
  • Jordan got to football practice at 7:05 p.m. Steve showed up 11 minutes later. What time did Steve get to practice?
  • Jack ran a marathon in 2 hours and 17 minutes. He crossed the finish line at 10:33 a.m. What time did the race start?
  • Marci was babysitting for her cousin. Her cousin was gone for 3 hours and 40 minutes. Marci left at 9:57 p.m. What time did she start babysitting? 
  • Caleb and his friends went to see a movie at 7:35 p.m. They left at 10:05 p.m. How long was the movie?
  • Francine got to work at 8:10 a.m. She left at 3:45 p.m. How long did Francine work?
  • Brandon went to bed at 9:15 p.m. It took him 23 minutes to fall asleep. What time did Brandon fall asleep?
  • Kelli had to wait in a long, slow-moving line to purchase a popular new video game that was just released. She got in line at 9:15 a.m. She left with the game at 11:07 a.m. How long did Kelli wait in line?
  • Jaydon went to batting practice Saturday morning at 8:30 a.m. He left at 11:42 a.m. How long was he at batting practice?
  • Ashton got behind on her reading assignment, so she had to read four chapters last night. She started at 8:05 p.m. and finished at 9:15 p.m. How long did it take Ashton to catch up on her assignment?
  • Natasha has a dentist appointment at 10:40 a.m. It should last 35 minutes. What time will she finish?
  • Mrs. Kennedy’s 3rd-grade class is going to the aquarium on a field trip. They are scheduled to arrive at 9:10 a.m. and leave at 1:40 p.m. How long will they spend at the aquarium?

Elapsed Time Games

Try these games and activities at home to help your children practice elapsed time.

Daily Schedule

Let your children keep track of their schedule and ask them to figure the elapsed time for each activity. For example, how long did your child spend eating breakfast, reading, taking a bath, or playing video games?

How Long Will It Take?

Give your kids practice with elapsed time by encouraging them to figure out how long daily activities take. For example, the next time you order a pizza online or by phone, you'll probably be given an estimated delivery time. Use that information to create a word problem that's relevant to your child's life, such as, "It's 5:40 p.m. now and the pizza shop says the pizza will be here at 6:20 p.m. How long will it take for the pizza to arrive?"

Order a set of time dice from online retailers or teacher supply stores. The set contains two twelve-sided dice, one with numbers representing the hours and the other with numbers representing minutes. Take turns rolling the time dice with your child. Each player should roll twice, then calculate the elapsed time between the two resulting dice times. (A pencil and paper will come in handy, as you'll want to jot down the time of the first roll.)

Elapsed Time Word Problem Answers

  •  2 hours and 2 minutes
  • 6 hours and 34 minutes
  • 2 hours and 30 minutes
  • 7 hours and 35 minutes
  • 1 hour and 52 minutes
  • 3 hours and 12 minutes
  • 1 hour and 10 minutes
  • 4 hours and 30 minutes
  • How To Tell Time in Spanish
  • Sinking of the RMS Titanic
  • A Timeline of the Sinking of the Titanic
  • How to Tell Time in Italian
  • When Is the Spring Equinox?
  • Free Math Word Problem Worksheets for Fifth-Graders
  • World War II: Operation Ten-Go
  • Proportions Word Problems Worksheet: Answers and Explanations
  • Telling Time in French
  • Learn About the Munich Olympic Massacre
  • Fundamental Lessons for Telling the Time
  • Practice Multiplication Skills With Times Tables Worksheets
  • The Longest Day of the Year
  • Solving Problems Involving Distance, Rate, and Time
  • The Assassination of President William McKinley
  • The Attack on Pearl Harbor

Logo for BCcampus Open Publishing

Want to create or adapt books like this? Learn more about how Pressbooks supports open publishing practices.

Chapter 2: Linear Equations

2.7 Variation Word Problems

Direct variation problems.

There are many mathematical relations that occur in life. For instance, a flat commission salaried salesperson earns a percentage of their sales, where the more they sell equates to the wage they earn. An example of this would be an employee whose wage is 5% of the sales they make. This is a direct or a linear variation, which, in an equation, would look like:

[latex]\text{Wage }(x)=5\%\text{ Commission }(k)\text{ of Sales Completed }(y)[/latex]

[latex]x=ky[/latex]

A historical example of direct variation can be found in the changing measurement of pi, which has been symbolized using the Greek letter π since the mid 18th century. Variations of historical π calculations are Babylonian [latex]\left(\dfrac{25}{8}\right),[/latex] Egyptian [latex]\left(\dfrac{16}{9}\right)^2,[/latex] and Indian [latex]\left(\dfrac{339}{108}\text{ and }10^{\frac{1}{2}}\right).[/latex] In the 5th century, Chinese mathematician Zu Chongzhi calculated the value of π to seven decimal places (3.1415926), representing the most accurate value of π for over 1000 years.

Pi is found by taking any circle and dividing the circumference of the circle by the diameter, which will always give the same value: 3.14159265358979323846264338327950288419716… (42 decimal places). Using an infinite-series exact equation has allowed computers to calculate π to 10 13 decimals.

[latex]\begin{array}{c} \text{Circumference }(c)=\pi \text{ times the diameter }(d) \\ \\ \text{or} \\ \\ c=\pi d \end{array}[/latex]

All direct variation relationships are verbalized in written problems as a direct variation or as directly proportional and take the form of straight line relationships. Examples of direct variation or directly proportional equations are:

  • [latex]x[/latex] varies directly as [latex]y[/latex]
  • [latex]x[/latex] varies as [latex]y[/latex]
  • [latex]x[/latex] varies directly proportional to [latex]y[/latex]
  • [latex]x[/latex] is proportional to [latex]y[/latex]
  • [latex]x[/latex] varies directly as the square of [latex]y[/latex]
  • [latex]x[/latex] varies as [latex]y[/latex] squared
  • [latex]x[/latex] is proportional to the square of [latex]y[/latex]
  • [latex]x[/latex] varies directly as the cube of [latex]y[/latex]
  • [latex]x[/latex] varies as [latex]y[/latex] cubed
  • [latex]x[/latex] is proportional to the cube of [latex]y[/latex]
  • [latex]x[/latex] varies directly as the square root of [latex]y[/latex]
  • [latex]x[/latex] varies as the root of [latex]y[/latex]
  • [latex]x[/latex] is proportional to the square root of [latex]y[/latex]

Example 2.7.1

Find the variation equation described as follows:

The surface area of a square surface [latex](A)[/latex] is directly proportional to the square of either side [latex](x).[/latex]

[latex]\begin{array}{c} \text{Area }(A) =\text{ constant }(k)\text{ times side}^2\text{ } (x^2) \\ \\ \text{or} \\ \\ A=kx^2 \end{array}[/latex]

Example 2.7.2

When looking at two buildings at the same time, the length of the buildings’ shadows [latex](s)[/latex] varies directly as their height [latex](h).[/latex] If a 5-story building has a 20 m long shadow, how many stories high would a building that has a 32 m long shadow be?

The equation that describes this variation is:

[latex]h=kx[/latex]

Breaking the data up into the first and second parts gives:

[latex]\begin{array}{ll} \begin{array}{rrl} \\ &&\textbf{1st Data} \\ s&=&20\text{ m} \\ h&=&5\text{ stories} \\ k&=&\text{find 1st} \\ \\ &&\text{Find }k\text{:} \\ h&=&kx \\ 5\text{ stories}&=&k\text{ (20 m)} \\ k&=&5\text{ stories/20 m}\\ k&=&0.25\text{ story/m} \end{array} & \hspace{0.5in} \begin{array}{rrl} &&\textbf{2nd Data} \\ s&=&\text{32 m} \\ h&=&\text{find 2nd} \\ k&=&0.25\text{ story/m} \\ \\ &&\text{Find }h\text{:} \\ h&=&kx \\ h&=&(0.25\text{ story/m})(32\text{ m}) \\ h&=&8\text{ stories} \end{array} \end{array}[/latex]

Inverse Variation Problems

Inverse variation problems are reciprocal relationships. In these types of problems, the product of two or more variables is equal to a constant. An example of this comes from the relationship of the pressure [latex](P)[/latex] and the volume [latex](V)[/latex] of a gas, called Boyle’s Law (1662). This law is written as:

[latex]\begin{array}{c} \text{Pressure }(P)\text{ times Volume }(V)=\text{ constant} \\ \\ \text{ or } \\ \\ PV=k \end{array}[/latex]

Written as an inverse variation problem, it can be said that the pressure of an ideal gas varies as the inverse of the volume or varies inversely as the volume. Expressed this way, the equation can be written as:

[latex]P=\dfrac{k}{V}[/latex]

Another example is the historically famous inverse square laws. Examples of this are the force of gravity [latex](F_{\text{g}}),[/latex] electrostatic force [latex](F_{\text{el}}),[/latex] and the intensity of light [latex](I).[/latex] In all of these measures of force and light intensity, as you move away from the source, the intensity or strength decreases as the square of the distance.

In equation form, these look like:

[latex]F_{\text{g}}=\dfrac{k}{d^2}\hspace{0.25in} F_{\text{el}}=\dfrac{k}{d^2}\hspace{0.25in} I=\dfrac{k}{d^2}[/latex]

These equations would be verbalized as:

  • The force of gravity [latex](F_{\text{g}})[/latex] varies inversely as the square of the distance.
  • Electrostatic force [latex](F_{\text{el}})[/latex] varies inversely as the square of the distance.
  • The intensity of a light source [latex](I)[/latex] varies inversely as the square of the distance.

All inverse variation relationship are verbalized in written problems as inverse variations or as inversely proportional. Examples of inverse variation or inversely proportional equations are:

  • [latex]x[/latex] varies inversely as [latex]y[/latex]
  • [latex]x[/latex] varies as the inverse of [latex]y[/latex]
  • [latex]x[/latex] varies inversely proportional to [latex]y[/latex]
  • [latex]x[/latex] is inversely proportional to [latex]y[/latex]
  • [latex]x[/latex] varies inversely as the square of [latex]y[/latex]
  • [latex]x[/latex] varies inversely as [latex]y[/latex] squared
  • [latex]x[/latex] is inversely proportional to the square of [latex]y[/latex]
  • [latex]x[/latex] varies inversely as the cube of [latex]y[/latex]
  • [latex]x[/latex] varies inversely as [latex]y[/latex] cubed
  • [latex]x[/latex] is inversely proportional to the cube of [latex]y[/latex]
  • [latex]x[/latex] varies inversely as the square root of [latex]y[/latex]
  • [latex]x[/latex] varies as the inverse root of [latex]y[/latex]
  • [latex]x[/latex] is inversely proportional to the square root of [latex]y[/latex]

Example 2.7.3

The force experienced by a magnetic field [latex](F_{\text{b}})[/latex] is inversely proportional to the square of the distance from the source [latex](d_{\text{s}}).[/latex]

[latex]F_{\text{b}} = \dfrac{k}{{d_{\text{s}}}^2}[/latex]

Example 2.7.4

The time [latex](t)[/latex] it takes to travel from North Vancouver to Hope varies inversely as the speed [latex](v)[/latex] at which one travels. If it takes 1.5 hours to travel this distance at an average speed of 120 km/h, find the constant [latex]k[/latex] and the amount of time it would take to drive back if you were only able to travel at 60 km/h due to an engine problem.

[latex]t=\dfrac{k}{v}[/latex]

[latex]\begin{array}{ll} \begin{array}{rrl} &&\textbf{1st Data} \\ v&=&120\text{ km/h} \\ t&=&1.5\text{ h} \\ k&=&\text{find 1st} \\ \\ &&\text{Find }k\text{:} \\ k&=&tv \\ k&=&(1.5\text{ h})(120\text{ km/h}) \\ k&=&180\text{ km} \end{array} & \hspace{0.5in} \begin{array}{rrl} \\ \\ \\ &&\textbf{2nd Data} \\ v&=&60\text{ km/h} \\ t&=&\text{find 2nd} \\ k&=&180\text{ km} \\ \\ &&\text{Find }t\text{:} \\ t&=&\dfrac{k}{v} \\ \\ t&=&\dfrac{180\text{ km}}{60\text{ km/h}} \\ \\ t&=&3\text{ h} \end{array} \end{array}[/latex]

Joint or Combined Variation Problems

In real life, variation problems are not restricted to single variables. Instead, functions are generally a combination of multiple factors. For instance, the physics equation quantifying the gravitational force of attraction between two bodies is:

[latex]F_{\text{g}}=\dfrac{Gm_1m_2}{d^2}[/latex]

  • [latex]F_{\text{g}}[/latex] stands for the gravitational force of attraction
  • [latex]G[/latex] is Newton’s constant, which would be represented by [latex]k[/latex] in a standard variation problem
  • [latex]m_1[/latex] and [latex]m_2[/latex] are the masses of the two bodies
  • [latex]d^2[/latex] is the distance between the centres of both bodies

To write this out as a variation problem, first state that the force of gravitational attraction [latex](F_{\text{g}})[/latex] between two bodies is directly proportional to the product of the two masses [latex](m_1, m_2)[/latex] and inversely proportional to the square of the distance [latex](d)[/latex] separating the two masses. From this information, the necessary equation can be derived. All joint variation relationships are verbalized in written problems as a combination of direct and inverse variation relationships, and care must be taken to correctly identify which variables are related in what relationship.

Example 2.7.5

The force of electrical attraction [latex](F_{\text{el}})[/latex] between two statically charged bodies is directly proportional to the product of the charges on each of the two objects [latex](q_1, q_2)[/latex] and inversely proportional to the square of the distance [latex](d)[/latex] separating these two charged bodies.

[latex]F_{\text{el}}=\dfrac{kq_1q_2}{d^2}[/latex]

Solving these combined or joint variation problems is the same as solving simpler variation problems.

First, decide what equation the variation represents. Second, break up the data into the first data given—which is used to find [latex]k[/latex]—and then the second data, which is used to solve the problem given. Consider the following joint variation problem.

Example 2.7.6

[latex]y[/latex] varies jointly with [latex]m[/latex] and [latex]n[/latex] and inversely with the square of [latex]d[/latex]. If [latex]y = 12[/latex] when [latex]m = 3[/latex], [latex]n = 8[/latex], and [latex]d = 2,[/latex] find the constant [latex]k[/latex], then use [latex]k[/latex] to find [latex]y[/latex] when [latex]m=-3[/latex], [latex]n = 18[/latex], and [latex]d = 3[/latex].

[latex]y=\dfrac{kmn}{d^2}[/latex]

[latex]\begin{array}{ll} \begin{array}{rrl} \\ \\ \\ && \textbf{1st Data} \\ y&=&12 \\ m&=&3 \\ n&=&8 \\ d&=&2 \\ k&=&\text{find 1st} \\ \\ &&\text{Find }k\text{:} \\ y&=&\dfrac{kmn}{d^2} \\ \\ 12&=&\dfrac{k(3)(8)}{(2)^2} \\ \\ k&=&\dfrac{12(2)^2}{(3)(8)} \\ \\ k&=& 2 \end{array} & \hspace{0.5in} \begin{array}{rrl} &&\textbf{2nd Data} \\ y&=&\text{find 2nd} \\ m&=&-3 \\ n&=&18 \\ d&=&3 \\ k&=&2 \\ \\ &&\text{Find }y\text{:} \\ y&=&\dfrac{kmn}{d^2} \\ \\ y&=&\dfrac{(2)(-3)(18)}{(3)^2} \\ \\ y&=&12 \end{array} \end{array}[/latex]

For questions 1 to 12, write the formula defining the variation, including the constant of variation [latex](k).[/latex]

  • [latex]x[/latex] is jointly proportional to [latex]y[/latex] and [latex]z[/latex]
  • [latex]x[/latex] varies jointly as [latex]z[/latex] and [latex]y[/latex]
  • [latex]x[/latex] is jointly proportional with the square of [latex]y[/latex] and the square root of [latex]z[/latex]
  • [latex]x[/latex] is inversely proportional to [latex]y[/latex] to the sixth power
  • [latex]x[/latex] is jointly proportional with the cube of [latex]y[/latex] and inversely to the square root of [latex]z[/latex]
  • [latex]x[/latex] is inversely proportional with the square of [latex]y[/latex] and the square root of [latex]z[/latex]
  • [latex]x[/latex] varies jointly as [latex]z[/latex] and [latex]y[/latex] and is inversely proportional to the cube of [latex]p[/latex]
  • [latex]x[/latex] is inversely proportional to the cube of [latex]y[/latex] and square of [latex]z[/latex]

For questions 13 to 22, find the formula defining the variation and the constant of variation [latex](k).[/latex]

  • If [latex]A[/latex] varies directly as [latex]B,[/latex] find [latex]k[/latex] when [latex]A=15[/latex] and [latex]B=5.[/latex]
  • If [latex]P[/latex] is jointly proportional to [latex]Q[/latex] and [latex]R,[/latex] find [latex]k[/latex] when [latex]P=12, Q=8[/latex] and [latex]R=3.[/latex]
  • If [latex]A[/latex] varies inversely as [latex]B,[/latex] find [latex]k[/latex] when [latex]A=7[/latex] and [latex]B=4.[/latex]
  • If [latex]A[/latex] varies directly as the square of [latex]B,[/latex] find [latex]k[/latex] when [latex]A=6[/latex] and [latex]B=3.[/latex]
  • If [latex]C[/latex] varies jointly as [latex]A[/latex] and [latex]B,[/latex] find [latex]k[/latex] when [latex]C=24, A=3,[/latex] and [latex]B=2.[/latex]
  • If [latex]Y[/latex] is inversely proportional to the cube of [latex]X,[/latex] find [latex]k[/latex] when [latex]Y=54[/latex] and [latex]X=3.[/latex]
  • If [latex]X[/latex] is directly proportional to [latex]Y,[/latex] find [latex]k[/latex] when [latex]X=12[/latex] and [latex]Y=8.[/latex]
  • If [latex]A[/latex] is jointly proportional with the square of [latex]B[/latex] and the square root of [latex]C,[/latex] find [latex]k[/latex] when [latex]A=25, B=5[/latex] and [latex]C=9.[/latex]
  • If [latex]y[/latex] varies jointly with [latex]m[/latex] and the square of [latex]n[/latex] and inversely with [latex]d,[/latex] find [latex]k[/latex] when [latex]y=10, m=4, n=5,[/latex] and [latex]d=6.[/latex]
  • If [latex]P[/latex] varies directly as [latex]T[/latex] and inversely as [latex]V,[/latex] find [latex]k[/latex] when [latex]P=10, T=250,[/latex] and [latex]V=400.[/latex]

For questions 23 to 37, solve each variation word problem.

  • The electrical current [latex]I[/latex] (in amperes, A) varies directly as the voltage [latex](V)[/latex] in a simple circuit. If the current is 5 A when the source voltage is 15 V, what is the current when the source voltage is 25 V?
  • The current [latex]I[/latex] in an electrical conductor varies inversely as the resistance [latex]R[/latex] (in ohms, Ω) of the conductor. If the current is 12 A when the resistance is 240 Ω, what is the current when the resistance is 540 Ω?
  • Hooke’s law states that the distance [latex](d_s)[/latex] that a spring is stretched supporting a suspended object varies directly as the mass of the object [latex](m).[/latex] If the distance stretched is 18 cm when the suspended mass is 3 kg, what is the distance when the suspended mass is 5 kg?
  • The volume [latex](V)[/latex] of an ideal gas at a constant temperature varies inversely as the pressure [latex](P)[/latex] exerted on it. If the volume of a gas is 200 cm 3 under a pressure of 32 kg/cm 2 , what will be its volume under a pressure of 40 kg/cm 2 ?
  • The number of aluminum cans [latex](c)[/latex] used each year varies directly as the number of people [latex](p)[/latex] using the cans. If 250 people use 60,000 cans in one year, how many cans are used each year in a city that has a population of 1,000,000?
  • The time [latex](t)[/latex] required to do a masonry job varies inversely as the number of bricklayers [latex](b).[/latex] If it takes 5 hours for 7 bricklayers to build a park wall, how much time should it take 10 bricklayers to complete the same job?
  • The wavelength of a radio signal (λ) varies inversely as its frequency [latex](f).[/latex] A wave with a frequency of 1200 kilohertz has a length of 250 metres. What is the wavelength of a radio signal having a frequency of 60 kilohertz?
  • The number of kilograms of water [latex](w)[/latex] in a human body is proportional to the mass of the body [latex](m).[/latex] If a 96 kg person contains 64 kg of water, how many kilograms of water are in a 60 kg person?
  • The time [latex](t)[/latex] required to drive a fixed distance [latex](d)[/latex] varies inversely as the speed [latex](v).[/latex] If it takes 5 hours at a speed of 80 km/h to drive a fixed distance, what speed is required to do the same trip in 4.2 hours?
  • The volume [latex](V)[/latex] of a cone varies jointly as its height [latex](h)[/latex] and the square of its radius [latex](r).[/latex] If a cone with a height of 8 centimetres and a radius of 2 centimetres has a volume of 33.5 cm 3 , what is the volume of a cone with a height of 6 centimetres and a radius of 4 centimetres?
  • The centripetal force [latex](F_{\text{c}})[/latex] acting on an object varies as the square of the speed [latex](v)[/latex] and inversely to the radius [latex](r)[/latex] of its path. If the centripetal force is 100 N when the object is travelling at 10 m/s in a path or radius of 0.5 m, what is the centripetal force when the object’s speed increases to 25 m/s and the path is now 1.0 m?
  • The maximum load [latex](L_{\text{max}})[/latex] that a cylindrical column with a circular cross section can hold varies directly as the fourth power of the diameter [latex](d)[/latex] and inversely as the square of the height [latex](h).[/latex] If an 8.0 m column that is 2.0 m in diameter will support 64 tonnes, how many tonnes can be supported by a column 12.0 m high and 3.0 m in diameter?
  • The volume [latex](V)[/latex] of gas varies directly as the temperature [latex](T)[/latex] and inversely as the pressure [latex](P).[/latex] If the volume is 225 cc when the temperature is 300 K and the pressure is 100 N/cm 2 , what is the volume when the temperature drops to 270 K and the pressure is 150 N/cm 2 ?
  • The electrical resistance [latex](R)[/latex] of a wire varies directly as its length [latex](l)[/latex] and inversely as the square of its diameter [latex](d).[/latex] A wire with a length of 5.0 m and a diameter of 0.25 cm has a resistance of 20 Ω. Find the electrical resistance in a 10.0 m long wire having twice the diameter.
  • The volume of wood in a tree [latex](V)[/latex] varies directly as the height [latex](h)[/latex] and the diameter [latex](d).[/latex] If the volume of a tree is 377 m 3 when the height is 30 m and the diameter is 2.0 m, what is the height of a tree having a volume of 225 m 3 and a diameter of 1.75 m?

Answer Key 2.7

Intermediate Algebra Copyright © 2020 by Terrance Berg is licensed under a Creative Commons Attribution-NonCommercial-ShareAlike 4.0 International License , except where otherwise noted.

Share This Book

problem solving involving word problems

OML Search

Word Problems Involving Subtraction

Mathway Calculator Widget

We welcome your feedback, comments and questions about this site or page. Please submit your feedback or enquiries via our Feedback page.

ExperTuition

Excellence Through Additional Support

5 Strategies to Teach Multi-Step Word Problems: Teacher’s Guide

strategies for teaching multistep word problems

Hey there, Teacher! Are you ready to empower your students with the skills they need to conquer multi-step word problems? Look no further! This comprehensive blog post is your go-to resource for effective strategies that will transform your classroom into a problem-solving powerhouse. Let’s dive in and unleash your students’ problem-solving potential!

Table of Contents

Word Problems

Word problems are an essential part of math education, as they help develop critical thinking and problem-solving skills.

Multi-step word problems, in particular, provide an excellent opportunity for students to apply their math skills in real-life scenarios.

Teaching students how to solve one-step word problems or multi-step word problems can be a complex task as it can be a challenging concept for most students to grasp.

However, with the right strategies and guidance, teachers can help their students become proficient problem solvers.

Strategies to Teach Multi-Step Word Problems

5 strategies for teaching multistep word problems

Now, let’s delve into the 5 strategies that teachers can employ to effectively teach multi-step word problem-solving to their students.

Model the Problem-Solving Process

Provide clear problem-solving strategies.

  • Provide Scaffolded Practice

Differentiate Instruction

  • Practice Regularly for Proficiency

One of the most effective ways to help students understand and solve multi-step word problems is by modeling the problem-solving process.

When teachers model the steps involved in solving a problem, students can observe and learn from their thinking and approach.

This approach helps students develop a deeper understanding of the problem and the strategies required to solve it.

modeling multistep word problem solving process

You can use the following approaches when modeling the problem-solving process.

Step-by-Step Approach

Storytelling approach, real-life examples.

When modeling the problem-solving process, it is crucial to take a step-by-step approach.

Break down the problem into smaller, manageable parts, and demonstrate how to tackle each part systematically.

By breaking down the problem, students can focus on one step at a time, reducing confusion and overwhelm.

Another effective strategy is to take a storytelling approach when presenting multi-step word problems.

Create narratives or scenarios around the problems, making them more interesting and captivating for students.

By presenting problems in a story format, students can visualize the context and relate to the characters or situations involved.

This approach enhances their problem-solving abilities and engages their imagination.

To make the modeling process more engaging and relatable, incorporate real-life examples into the multi-step word problems.

Relating the problems to situations that students encounter in their daily lives helps them connect with the content and see the practical applications of mathematical concepts.

For instance, you can present a word problem involving shopping or calculating distances during a trip.

As a teacher, your role is to equip your students with effective problem-solving strategies that will empower them to confidently tackle multi-step word problems.

strategies for solving multistep word problems

Here are some key multi-step problem-solving strategies to share with your students:

Read the Problem Carefully

Identify the question, plan and break down the problem, choose the correct operations, solve step-by-step, check and reflect.

The first step in solving a multi-step word problem is to read the problem carefully.

Emphasize the importance of taking the time to understand the given information.

Encourage your students to identify the essential details, underline or highlight important numbers or quantities.

By comprehending the problem thoroughly, students will lay a strong foundation for their problem-solving journey.

After understanding the given information, students should identify the question they need to answer.

Guide your students to identify the question or the unknown they need to solve.

Assist them in recognizing what information is missing and what they are being asked to find.

Encourage them to clearly define the question to maintain focus and direction throughout the problem-solving process.

Teach your students to develop a plan and break down the problem into smaller, more manageable steps.

By doing so, they can avoid feeling overwhelmed by the complexity of multi-step word problems.

Discuss possible approaches, such as drawing diagrams, making tables, or using equations.

By having a plan in place or by creating a roadmap for the problem-solving journey, students can proceed with confidence and avoid making unnecessary mistakes.

Guide your students in choosing the correct operations or mathematical strategies for each step of the problem.

Help them consider the problem’s context and the relationships between different quantities.

By selecting the appropriate operations, such as addition, subtraction, multiplication, or division, students can accurately solve each component of the multi-step word problem.

Encourage your students to solve the problem step-by-step, following the plan they devised earlier.

By solving each step individually and linking them together, students can see the overall problem as a series of interconnected components, making it easier to navigate and solve.

Remind students to show their work neatly and clearly. This approach not only helps students organize their thoughts but also allows you, as their teacher, to identify any errors or misconceptions they may have.

Provide guidance and support as needed throughout their problem-solving process.

After obtaining a solution, it’s essential that students check their answer.

Teach your students the importance of checking their work and reflecting on their solution.

Encourage them to evaluate whether their answer makes sense within the context of the problem. They can do this by revisiting the original problem, reapplying the given information, and verifying if the solution satisfies the question asked.

This step promotes critical thinking and helps students identify and correct any mistakes they may have made.

Scaffolded Practice

As a teacher, it is crucial to provide scaffolded practice to support your students in mastering multi-step word problems.

Scaffolded practice involves breaking down the problem-solving process into smaller, manageable steps and gradually increasing the complexity of the problems.

This approach allows students to build their skills incrementally and gain confidence as they progress.

strategies to implement scaffolded multistep word problem practice

Here are some strategies to implement scaffolded practice:

Start with Simple Problems

Provide guided practice, increase complexity gradually, use visual aids and models, break down complex problems, provide independent practice opportunities.

Begin by introducing your students to simple, one-step word problems .

These problems will help them understand the basics of problem-solving and build their confidence.

Provide clear explanations and model the problem-solving process step-by-step.

Reinforce the importance of reading the problem carefully and identifying the key information.

Guide your students through practice problems by solving them together as a class.

Discuss the steps involved and explain the reasoning behind each step.

Encourage students to ask questions and engage in discussions about problem-solving strategies.

Offer support and guidance as needed, ensuring that students understand each concept before moving on.

Gradually increase the complexity of the problems as your students become more comfortable with one-step word problems.

Introduce multistep word problems by adding another step or operation at a time.

Provide clear explanations of each step and emphasize the relationships between different quantities in the problem.

Encourage students to apply the problem-solving strategies they have learned.

Utilize visual aids and models to help students visualize the problem and understand the relationships between different quantities.

Use diagrams, charts, or manipulatives to represent the information given in the problem. This visual representation can enhance students’ understanding and support their problem-solving process.

For complex multi-step word problems, break them down into smaller components.

Guide students through each step, explaining the rationale behind each operation and encouraging them to show their work.

This step-by-step approach helps students tackle complex problems more effectively and reduces feelings of overwhelm.

Offer opportunities for students to practice independently.

Provide worksheets or online resources that offer a range of multi-step word problems at various difficulty levels.

Encourage students to work through the problems on their own, applying the strategies they have learned. Offer feedback and support as they progress.

As a teacher, it is essential to differentiate your instruction to meet the diverse needs of your students.

Differentiation allows you to tailor your teaching methods, materials, and assessments to accommodate the individual learning styles, abilities, and interests of your students.

strategies for differentiating multistep word problems instructions

Here are some strategies for differentiating instruction in multi-step word problem-solving:

Assess Readiness and Grouping

Offer multiple problem-solving approaches, vary the level of difficulty, flexible grouping strategies, accommodations and supports, formative assessments and feedback.

Assess the readiness of your students by evaluating their prior knowledge and skills related to multi-step word problems.

Group students based on their readiness levels, whether they require additional support or enrichment.

Provide targeted instruction and resources to address the specific needs of each group.

Recognize that students have different learning styles and preferences.

Offer a variety of problem-solving approaches to cater to their individual needs.

Provide visual representations, manipulatives, or verbal explanations to support visual, kinesthetic, or auditory learners.

Allow students to choose the method that best suits their learning style.

Recognize that students have different levels of proficiency in problem-solving.

Differentiate the level of difficulty in the problems you assign.

Provide additional challenge problems for high-achieving students and offer extra support or modified problems for those who may struggle.

Adjust the complexity of the problems to ensure that each student is appropriately challenged.

Implement flexible grouping strategies to allow students to learn from and support each other.

Arrange students in pairs or small groups based on their strengths and weaknesses.

Encourage collaborative problem-solving activities where students can share their strategies, explain their thinking, and learn from their peers.

Offer opportunities for peer tutoring or mentoring.

Recognize the diverse needs of students with learning disabilities or English language learners.

Provide accommodations and support to ensure their understanding and success.

Adapt the materials, provide visual cues or graphic organizers, offer additional time, or provide translated resources when necessary.

Collaborate with special education or language support specialists to address individual needs effectively.

Use formative assessments to gather ongoing feedback on students’ progress in multi-step word problem-solving.

Monitor their understanding, identify misconceptions, and provide timely feedback. Offer specific guidance and support based on individual needs.

Encourage students to reflect on their problem-solving strategies and provide self-assessment opportunities.

Practice Multi-Step Word Problems Regularly for Proficiency

It is important to emphasize the value of regular practice to help your students achieve proficiency in solving multi-step word problems.

Consistent practice not only reinforces problem-solving strategies and concepts but also builds confidence and fluency in applying them.

regular multistep word problem practice

Here are some strategies to promote regular practice:

Assign Regular Problem-Solving Exercises

  • Integrate Multistep Word Problems in Daily Lessons
  • Encourage Independent Practice

Offer Problem-Solving Discussions and Presentations

Assign regular problem-solving exercises as part of your students’ homework or in-class activities.

Provide a variety of multi-step word problems that cover different mathematical concepts and real-life scenarios.

Gradually increase the difficulty level of the problems to challenge your students and foster their growth.

Integrate Multi-Step Word Problems in Daily Lessons

Incorporate multi-step word problems into your daily math lessons. Introduce them as real-world applications of the mathematical concepts you are teaching.

Show your students how these problems relate to their everyday lives and the importance of problem-solving skills in various contexts.

Inspire active participation and engagement during problem-solving activities.

Encourage Independent Practice of Multi-Step Word Problems

independent multistep word problems practice

Encourage your students to practice independently outside of class.

Recommend math resources, such as textbooks, workbooks, or online platforms, that provide additional multi-step word problems for practice.

Encourage them to set aside dedicated time each week to work on problem-solving skills.

Remind them of the benefits of consistent practice in building proficiency.

solving multistep word problems discussions

Create opportunities for students to present and discuss their problem-solving strategies with the class.

Encourage them to explain their thinking, justify their solutions, and engage in constructive discussions.

This not only enhances their communication skills but also exposes them to different problem-solving approaches and fosters a collaborative learning environment.

Recommended Materials for Multi-Step Word Problems Practice

multistep word problems worksheets and task cards

To enhance your students’ practice in solving multi-step word problems, consider incorporating the following recommended materials:

Math Workbooks

Online problem-solving platforms, math games and activities, task cards and worksheets.

Math workbooks provide a structured approach to practicing multi-step word problems.

Look for workbooks that specifically focus on problem-solving skills and offer a variety of problem types and difficulty levels.

These workbooks typically include step-by-step explanations and examples, providing students with opportunities to apply their problem-solving strategies independently.

Encourage your students to work through the problems in the workbooks, highlighting the importance of showing their work and explaining their reasoning.

You can assign specific pages or problem sets from the workbook as part of their regular practice routine.

Leverage the power of technology by using online problem-solving platforms.

These platforms offer interactive multi-step word problems that engage students and provide immediate feedback.

Look for platforms that align with your curriculum and allow you to track students’ progress.

Online problem-solving platforms often provide a range of difficulty levels and adaptive features that adjust the complexity of the problems based on individual performance.

Encourage your students to explore these platforms during their independent practice time or assign specific problems for them to solve online.

Engage your students in learning through math games and activities.

These interactive and hands-on experiences make practice enjoyable and foster a positive attitude towards problem-solving.

Look for math games and activities that specifically focus on multi-step word problems.

You can create your own games using task cards or find existing games that align with your curriculum.

Set up problem-solving stations where students rotate and solve different multi-step word problems in a game format.

These activities promote collaboration, critical thinking, and a deeper understanding of problem-solving strategies.

Task cards and worksheets provide targeted practice opportunities for multi-step word problems.

While worksheets offer a collection of problems on a single sheet, task cards typically consist of individual problems on small cards.

These resources allow for flexibility in assigning practice and can be tailored to the specific needs of your students.

Select task cards or worksheets that align with the topics and skills you want your students to practice.

Consider using a variety of formats, such as multiple-choice, open-ended, or guided questions, to cater to different learning preferences.

Provide clear instructions and encourage students to work through the problems independently or in small groups.

If you’re looking for engaging worksheets and task cards for multi-step word problems to use with your students, then my differentiated worksheets and task cards will be a perfect fit for you. You can find them at my resources stores below:

  • Website shop
  • Made by Teachers store

Congratulations, teachers! You have now equipped yourself with a toolkit of effective strategies and resources to teach multi-step word problems with confidence.

By providing clear problem-solving strategies, implementing scaffolded practice, differentiating your instruction, and promoting regular practice, you are setting your students up for success.

Embrace the journey of teaching and learning multi-step word problems, celebrate the progress of your students, and watch as they develop into proficient problem solvers.

Remember, with your guidance and support, their potential knows no bounds!

how to teach multistep word problems

Multistep Word Problems – FAQs

What are two-step and multi-step problems.

Two-step and multi-step problems are types of word problems that require multiple mathematical operations to be performed to find the solution. In two-step problems, students need to apply two operations, such as addition and subtraction or multiplication and division, to solve the problem. Multi-step problems may involve more than two operations and often require students to perform a series of steps to arrive at the final answer.

Why Are Multi-Step Word Problems So Important?

Multi-step word problems are important because they reflect real-world situations that students may encounter in their daily lives. By solving these problems, students develop critical thinking, analytical reasoning, and problem-solving skills. Multi-step word problems also help students apply mathematical concepts in context, promoting a deeper understanding of the subject matter.

Why do students struggle with multi-step word problems?

Students often struggle with multi-step word problems due to several reasons. First, these problems require higher-level thinking skills, such as analyzing and synthesizing information. Second, students may find it challenging to identify the relevant information and determine which operations to use. Additionally, students may struggle with organizing their thoughts and applying problem-solving strategies effectively.

What are the common errors in solving multi-step word problems?

Common errors in solving multi-step word problems include:

  • Misinterpreting the problem: Failing to understand the problem correctly and misidentifying the key information.
  • I ncorrect selection of operations: Choosing the wrong operations or using them in the wrong order.
  • Calculation errors: Making mistakes in performing arithmetic calculations.
  • Forgetting to check the answer: Neglecting to verify if the solution aligns with the question asked and the context of the problem.

DOWNLOAD FREEBIES

Please, enter your NON-WORK EMAIL to ensure that you get the freebies!

We don’t spam! Read our privacy policy for more info.

free 3rd 4th 5th grade math worksheets

Check your inbox or spam folder to confirm your subscription for the link to freebie library.

Related Posts

multiplication strategies for grade 4 and grade 5

Multiplication Strategies for Grade 4 and Grade 5

4th grade and 5th grade division strategies

Division Strategies for Grade 4 and Grade 5

Leave a comment cancel reply.

Your email address will not be published. Required fields are marked *

Save my name, email, and website in this browser for the next time I comment.

Teaching Methods for Solving Word Problems

In partnership with Model Teaching, an industry leader in supporting educators, this course focuses on addressing challenges students encounter in math and offers solutions through two different methods for solving word problems. Model Teaching’s Mission is to improve student performance by directly supporting teachers with quality content and resources. Participants will receive a downloadable PDF of the content, along with valuable graphic organizers and student resources to support implementation in the classroom.

Details + Objectives

Course code: t14751.

This teacher professional development course will teach you about the challenges students often encounter when solving word problems and present varied solutions for teaching problem-solving skills to your students. You will be provided with a detailed plan for teaching two different problem-solving strategies in your classroom, depending on your students' needs: the GUESS method and the CUBES method for word problems. The course also comes with a downloadable PDF of the course content, graphic organizers, and supporting student resources.

What you will learn

  • Evaluate the common challenges students face when solving word problems 
  • Analyze solutions teachers can consider when teaching word problem-solving 
  • Compare the CUBES method and the GUESS method to determine which is the best method to teach your students 

How you will benefit

  • More easily determine where a student struggles in solving a word problem by identifying the step they begin to struggle with 
  • Effectively communicate a clear path to completing word problems and give confidence to students who may feel that word problems present a daunting task 
  • Students will be able to apply their learning more easily to various word problems once they have mastered the strategies you teach 
  • Provide students with increasingly complex word problems as they begin to master strategies to solve them 

How the course is taught

  • Self-Guided, online course
  • 3 Months access
  • 1 course hours
  • The video and article will provide you with two specific methods for solving word problems and ideas for teaching your students these skills. You will learn about common strategies required for solving word problems and challenges that arise so you can help correct and support your students. The course also includes some “Reflect or Discuss” prompts to help you connect with the course content and ends with a “Try This Task” to guide you explicitly on how you might implement the ideas into the classroom. 
  • You will answer questions related to strategies for solving word problems. Quizzes are automatically scored and provide feedback on answer choice rationale. 
  • The reflection prompt requires you to plan for use of the “try this task” by either reflecting on the content yourself or discussing them with the colleague. You will then discuss a new concept you can attempt to implement in the future based on something you learned in the course. 
  • This short statement helps you reflect on your ideas and assess whether you might be successful in your implementation. 
  • Additional content suggestions are provided to enhance and expand your understanding of supporting students in word probably analysis.

Instructors & Support

Requirements.

Prerequisites:

There are no prerequisites to take this course. 

Requirements:

Hardware Requirements:

  • This course can be taken on either a PC, Mac, or Chromebook.

Software Requirements:

  • PC: Windows 10 or later.
  • Mac: macOS 10.6 or later.
  • Browser: The latest version of Google Chrome or Mozilla Firefox is preferred. Microsoft Edge and Safari are also compatible.
  • Microsoft Word Online
  • Editing of a Microsoft Word document is required in this course. You may use a free version of Microsoft Word Online, or Google Docs if you do not have Microsoft Office installed on your computer. Model Teaching can provide support for this.
  • Adobe Acrobat Reader
  • Software must be installed and fully operational before the course begins.
  • Email capabilities and access to a personal email account.

Instructional Material Requirements:

The instructional materials required for this course are included in enrollment and will be available online.

When can I get started?

Your course begins immediately after you enroll.​

How does it work?

You have 3 months of access to the course. After enrolling, you can learn and complete the course at your own pace, within the allotted access period. You will have the opportunity to interact with other students in the online discussion area.

How long do I have to complete each lesson?

There is no time limit to complete each lesson, other than completing all lessons within the allotted access period. Discussion areas for each lesson are open for the entire duration of the course.

What if I need an extension?

Because this course is self-guided, no extensions will be granted after the start of your enrollment.

Related Courses

Thank you for visiting nature.com. You are using a browser version with limited support for CSS. To obtain the best experience, we recommend you use a more up to date browser (or turn off compatibility mode in Internet Explorer). In the meantime, to ensure continued support, we are displaying the site without styles and JavaScript.

  • View all journals
  • Explore content
  • About the journal
  • Publish with us
  • Sign up for alerts
  • 20 March 2024
  • Correction 21 March 2024

Mathematician who tamed randomness wins Abel Prize

  • Davide Castelvecchi

You can also search for this author in PubMed   Google Scholar

You have full access to this article via your institution.

Michel Talagrand.

Michel Talagrand studies stochastic processes, mathematical models of phenomena that are governed by randomness. Credit: Peter Bagde/Typos1/Abel Prize 2024

A mathematician who developed formulas to make random processes more predictable and helped to solve an iconic model of complex phenomena has won the 2024 Abel Prize, one of the field’s most coveted awards. Michel Talagrand received the prize for his “contributions to probability theory and functional analysis, with outstanding applications in mathematical physics and statistics”, the Norwegian Academy of Science and Letters in Oslo announced on 20 March.

Assaf Naor, a mathematician at Princeton University in New Jersey, says it is difficult to overestimate the impact of Talagrand’s work. “There are papers posted maybe on a daily basis where the punchline is ‘now we use Talagrand’s inequalities’,” he says.

Talagrand’s reaction on hearing the news was incredulity. “There was a total blank in my mind for at least four seconds,” he says. “If I had been told an alien ship had landed in front of the White House, I would not have been more surprised.”

The Abel Prize was modelled after the Nobel Prizes — which do not include mathematics — and was first awarded in 2003. The recipient wins a sum of 7.5 million Norwegian kroner (US$700,000).

‘Like a piece of art’

Talagrand specializes in the theory of probability and stochastic processes, which are mathematical models of phenomena governed by randomness. A typical example is a river’s water level, which is highly variable and is affected by many independent factors, including rain, wind and temperature, Talagrand says. His proudest achievement was his inequalities 1 , a set of formulas that poses limits to the swings in stochastic processes. His formulas express how the contributions of many factors often cancel each other out — making the overall result less variable, not more.

“It’s like a piece of art,” says Abel-committee chair Helge Holden, a mathematician at the Norwegian University of Science and Technology in Trondheim. “The magic here is to find a good estimate, not just a rough estimate.”

problem solving involving word problems

Abel Prize: pioneer of ‘smooth’ physics wins top maths award

Thanks to Talagrand’s techniques, “many things that seem complicated and random turn out to be not so random”, says Naor. His estimates are extremely powerful, for example for studying problems such as optimizing the route of a delivery truck. Finding a perfect solution would require an exorbitant amount of computation, so computer scientists can instead calculate the lengths of a limited number of random candidate routes and then take the average — and Talagrand’s inequalities ensure that the result is close to optimal.

Talagrand also completed the solution to a problem posed by theoretical physicist Giorgio Parisi — work that ultimately helped Parisi to earn a Nobel Prize in Physics in 2021. In 1979, Parisi, now at the University of Rome, proposed a complete solution for the structure of a spin glass — an abstracted model of a material in which the magnetization of each atom tends to flip up or down depending on those of its neighbours.

Parisi’s arguments were rooted in his powerful intuition in physics, and followed steps that “mathematicians would consider as sorcery”, Talagrand says, such as taking n copies of a system — with n being a negative number. Many researchers doubted that Parisi’s proof could be made mathematically rigorous. But in the early 2000s, the problem was completely solved in two separate works, one by Talagrand 2 and an earlier one by Francesco Guerra 3 , a mathematical physicist who is also at the University of Rome.

Finding motivation

Talagrand’s journey to becoming a top researcher was unconventional. Born in Béziers, France, in 1952, he lost vision in his right eye at age five because of a genetic predisposition to detachment of the retina. Although while growing up in Lyon he was a voracious reader of popular science magazines, he struggled at school, particularly with the complex rules of French spelling. “I never really made peace with orthography,” he told an interviewer in 2019 .

His turning point came at age 15, when he received emergency treatment for another retinal detachment, this time in his left eye. He had to miss almost an entire year of school. The terrifying experience of nearly losing his sight — and his father’s efforts to keep his mind busy while his eyes were bandaged — gave Talagrand a renewed focus. He became a highly motivated student after his recovery, and began to excel in national maths competitions.

problem solving involving word problems

Just 5 women have won a top maths prize in the past 90 years

Still, Talagrand did not follow the typical path of gifted French students, which includes two years of preparatory school followed by a national admission competition for highly selective grandes écoles such as the École Normale Supérieure in Paris. Instead, he studied at the University of Lyon, France, and then went on to work as a full-time researcher at the national research agency CNRS, first in Lyon and later in Paris, where he spent more than a decade in an entry-level job. Apart from a brief stint in Canada, followed by a trip to the United States where he met his wife, he worked at the CNRS until his retirement.

Talagrand loves to challenge other mathematicians to solve problems that he has come up with — offering cash to those who do — and he keeps a list of those problems on his website. Some have been solved, leading to publications in major maths journals . The prizes come with some conditions: “I will award the prizes below as long as I am not too senile to understand the proofs I receive. If I can’t understand them, I will not pay.”

Nature 627 , 714-715 (2024)

doi: https://doi.org/10.1038/d41586-024-00839-6

Updates & Corrections

Correction 21 March 2024 : An earlier version of this article stated that Giorgio Parisi won the Nobel Prize in Physics in 2001. He in fact won in 2021.

Talagrand, M. Publ. Math. IHES 81 , 73–205, (1995).

Article   Google Scholar  

Talagrand, M. Ann. Math. 163 , 221–263 (2006).

Guerra, F. Commun. Math. Phys. 233 , 1–12 (2003).

Download references

Reprints and permissions

Related Articles

problem solving involving word problems

Fermat's last theorem earns Andrew Wiles the Abel Prize

'Beautiful mind' John Nash adds Abel Prize to his Nobel

  • Mathematics and computing

A global timekeeping problem postponed by global warming

A global timekeeping problem postponed by global warming

Article 27 MAR 24

Climate change has slowed Earth’s rotation — and could affect how we keep time

Climate change has slowed Earth’s rotation — and could affect how we keep time

News 27 MAR 24

High-threshold and low-overhead fault-tolerant quantum memory

High-threshold and low-overhead fault-tolerant quantum memory

Tenure-track Assistant Professor in Ecological and Evolutionary Modeling

Tenure-track Assistant Professor in Ecosystem Ecology linked to IceLab’s Center for modeling adaptive mechanisms in living systems under stress

Umeå, Sweden

Umeå University

problem solving involving word problems

Faculty Positions in Westlake University

Founded in 2018, Westlake University is a new type of non-profit research-oriented university in Hangzhou, China, supported by public a...

Hangzhou, Zhejiang, China

Westlake University

problem solving involving word problems

Postdoctoral Fellowships-Metabolic control of cell growth and senescence

Postdoctoral positions in the team Cell growth control by nutrients at Inst. Necker, Université Paris Cité, Inserm, Paris, France.

Paris, Ile-de-France (FR)

Inserm DR IDF Paris Centre Nord

problem solving involving word problems

Zhejiang Provincial Hospital of Chinese Medicine on Open Recruitment of Medical Talents and Postdocs

Director of Clinical Department, Professor, Researcher, Post-doctor

The First Affiliated Hospital of Zhejiang Chinese Medical University

problem solving involving word problems

Sir Run Run Shaw Hospital, School of Medicine, Zhejiang University, Warmly Welcomes Talents Abroad

“Qiushi” Distinguished Scholar, Zhejiang University, including Professor and Physician

No. 3, Qingchun East Road, Hangzhou, Zhejiang (CN)

Sir Run Run Shaw Hospital Affiliated with Zhejiang University School of Medicine

problem solving involving word problems

Sign up for the Nature Briefing newsletter — what matters in science, free to your inbox daily.

Quick links

  • Explore articles by subject
  • Guide to authors
  • Editorial policies

Home

Reading & Math for K-5

Easter Worksheets

  • Kindergarten
  • Learning numbers
  • Comparing numbers
  • Place Value
  • Roman numerals
  • Subtraction
  • Multiplication
  • Order of operations
  • Drills & practice
  • Measurement
  • Factoring & prime factors
  • Proportions
  • Shape & geometry
  • Data & graphing
  • Word problems
  • Children's stories
  • Leveled Stories
  • Context clues
  • Cause & effect
  • Compare & contrast
  • Fact vs. fiction
  • Fact vs. opinion
  • Main idea & details
  • Story elements
  • Conclusions & inferences
  • Sounds & phonics
  • Words & vocabulary
  • Reading comprehension
  • Early writing
  • Numbers & counting
  • Simple math
  • Social skills
  • Other activities
  • Dolch sight words
  • Fry sight words
  • Multiple meaning words
  • Prefixes & suffixes
  • Vocabulary cards
  • Other parts of speech
  • Punctuation
  • Capitalization
  • Narrative writing
  • Opinion writing
  • Informative writing
  • Cursive alphabet
  • Cursive letters
  • Cursive letter joins
  • Cursive words
  • Cursive sentences
  • Cursive passages
  • Grammar & Writing

Breadcrumbs

  • Word Problems
  • Add/subtract fractions

Math Workbooks for Grade 5

Download & Print From only $2.60

Add & subtract fractions word problems

Like & unlike denominators.

Below are our grade 5 math word problem worksheet on adding and subtracting fractions.  The problems include both like and unlike denominators , and may include more than two terms.

problem solving involving word problems

These worksheets are available to members only.

Join K5 to save time, skip ads and access more content. Learn More

More word problem worksheets

Explore all of our math word problem worksheets , from kindergarten through grade 5.

What is K5?

K5 Learning offers free worksheets , flashcards  and inexpensive  workbooks  for kids in kindergarten to grade 5. Become a member  to access additional content and skip ads.

Our members helped us give away millions of worksheets last year.

We provide free educational materials to parents and teachers in over 100 countries. If you can, please consider purchasing a membership ($24/year) to support our efforts.

Members skip ads and access exclusive features.

Learn about member benefits

This content is available to members only.

  • Forgot Password?

IMAGES

  1. Solving Word Problems Chart Grade 2-8

    problem solving involving word problems

  2. Solving Word Problems Involving Linear Equations Math Worksheets

    problem solving involving word problems

  3. Solving Word Problems Involving Perimeter Area of Rectangle Worksheets

    problem solving involving word problems

  4. Decimals Word Problems Worksheet

    problem solving involving word problems

  5. 5 Unique Word Problem Solving Strategies That Get Results

    problem solving involving word problems

  6. Problem Solving Addition Word Problems For Grade 2 Worksheets

    problem solving involving word problems

VIDEO

  1. Solving Problems Involving Quadratic Equations

  2. Solving Word Problems Involving Quadratic Inequalities

  3. I Problem Solving involving Simultaneous Linear Equations Q 5 6 7

  4. I Problem Solving involving Simultaneous Linear Equations Q 28 c

  5. Solving a Word Problem Involving Rates and Time Conversion

  6. I Problem Solving involving Simultaneous Linear Equations Q 16 17 18 19

COMMENTS

  1. 120 Math Word Problems To Challenge Students Grades 1 to 8

    Feature Red Herrings: Including unnecessary information introduces another problem-solving element, overwhelming many elementary students. A key to differentiated instruction, word problems that students can relate to and contextualize will capture interest more than generic and abstract ones. Final thoughts about math word problems

  2. Word Problems Calculator

    An age problem is a type of word problem in math that involves calculating the age of one or more people at a specific point in time. These problems often use phrases such as 'x years ago,' 'in y years,' or 'y years later,' which indicate that the problem is related to time and age.

  3. 10 Best Strategies for Solving Math Word Problems

    A Guide on Steps to Solving Word Problems: 10 Strategies. 1. Understand the Problem by Paraphrasing. One of the first steps in tackling a math word problem is to make sure your students understand what the problem is asking. Encourage them to paraphrase the problem in their own words.

  4. Apply a Problem-Solving Strategy to Word Problems

    Step 5. Solve the equation using good algebra techniques. Even if you know the answer right away, using algebra will better prepare you to solve problems that do not have obvious answers. Write the equation. 18= 1 2p 18 = 1 2 p. Multiply both sides by 2. 2⋅18=2⋅ 1 2p 2 ⋅ 18 = 2 ⋅ 1 2 p. Simplify. 36=p 36 = p.

  5. Apply a Problem-Solving Strategy to Basic Word Problems

    Step 5. Solve the equation using good algebra techniques. Even if you know the answer right away, using algebra will better prepare you to solve problems that do not have obvious answers. Write the equation. [Math Processing Error] 18 = 1 2 p. Multiply both sides by 2. [Math Processing Error] 2 ⋅ 18 = 2 ⋅ 1 2 p.

  6. Math Word Problems

    For example, students may need a way to figure out what 7 × 8 is or have previously memorized the answer before you give them a word problem that involves finding the answer to 7 × 8. There are a number of strategies used in solving math word problems; if you don't have a favorite, try the Math-Drills.com problem-solving strategy:

  7. 1.20: Word Problems for Linear Equations

    Solution: Translating the problem into an algebraic equation gives: 2x − 5 = 13. We solve this for x. First, add 5 to both sides. 2x = 13 + 5, so that 2x = 18. Dividing by 2 gives x = 18 2 = 9. c) A number subtracted from 9 is equal to 2 times the number. Find the number. Solution: We translate the problem to algebra.

  8. Algebraic word problems

    Solving algebraic word problems requires us to combine our ability to create equations and solve them. To solve an algebraic word problem: Define a variable. Write an equation using the variable. Solve the equation. If the variable is not the answer to the word problem, use the variable to calculate the answer.

  9. Challenge Exercises Integer Word Problems

    Challenge Exercises Integer Word Problems. Directions: Read each question below. Click once in an ANSWER BOX and type in your answer; then click ENTER. After you click ENTER, a message will appear in the RESULTS BOX to indicate whether your answer is correct or incorrect. To start over, click CLEAR. Each answer should be given as a positive or ...

  10. 20 Word Problems For Year 6: Develop Problem Solving Skills

    Solving problems involving comparing, ordering and sequencing fractions; using the four operations and problems which require answers to be rounded to specified degrees of accuracy. ... Word problems in SATS. Problem solving is an important part of the preparation for the Key Stage 2 SATs, as this is the area children tend to find the most ...

  11. Algebraic Expressions From Word Problems

    In some cases, an algebraic expression can also be expressed using parentheses. These problems can be simple as: Example 1: The sum of a number and 3 is equal to 6. What is that number? This ...

  12. Word Problems

    Sharpen your problem-solving skills with fun exercises based on real-world scenarios. Start for free! ... Help your child solve word problems on adding fractions & mixed numbers. 3 4 5. VIEW DETAILS. ... Solidify your math skills by practicing addition problems involving mixed numbers. 3 4 5. VIEW DETAILS. Addition and Subtraction Word Problems.

  13. GCF and LCM Word Problems

    These word problems need the use of greatest common factors (GCFs) or least common multiples (LCMs) to solve. Mixing GCF and LCM word problems encourages students to read and think about the questions, rather than simply recognizing a pattern to the solutions. Worksheet #1 Worksheet #2 Worksheet #3 Worksheet #4. Worksheet #5 Worksheet #6. Similar:

  14. Radical Equation Word Problems

    Example walk through: word problem involving radicals. The key to word problems is in the translation. Let's walk through the problem below and solve it. Lori recently purchased a square painting with an area of 780 cm2. How much wood does Lori need to make the frame? Don't worry about the width and height of the wood, just find the length.

  15. Word Problems on Linear Equations

    Step-by-step application of linear equations to solve practical word problems: 1. The sum of two numbers is 25. One of the numbers exceeds the other by 9. Find the numbers. Let the number be x. Therefore, the two numbers are 8 and 17. 2.The difference between the two numbers is 48. The ratio of the two numbers is 7:3.

  16. PDF 7. Word PROBLEMS WITH INTEGERS

    12. The temperature was -3o C last night. It is now -4o C. What was the change in temperature? 13. While watching a football game, Lin Chow decided to list yardage gained as positive integers and yardage lost as negative integers. After these plays, Lin recorded 14, -7, and 9.

  17. Solving Functions from Word Problems

    f(x) = 0.75(x − 1) + 5. Correct answer: f(x) = 0.75(x − 1) + 5. Explanation: Notice that the question describes a linear equation because there is a constant rate of change (the cost per topping). This means we can use slope intercept form to describe the scenario. Recall that slope intercept form is. y = mx + b.

  18. 20 Elapsed Time Word Problems

    20 Elapsed Time Word Problems. Elapsed time is the amount of time that passes between the beginning and the end of an event. The concept of elapsed time fits nicely in the elementary school curriculum. Beginning in third grade, students should be able to tell and write time to the nearest minute and solve word problems involving addition and ...

  19. Word Problems with Fractions

    The solution to this problem is an irreducible fraction (a fraction which cannot be simplified). Therefore, there is nothing left to do. Word problems with fractions: involving two fractions. In these problems, we should remember how to carry out operations with fractions. Carefully read the following problem and the steps we have taken to ...

  20. 2.7 Variation Word Problems

    Consider the following joint variation problem. Example 2.7.6. y y varies jointly with m m and n n and inversely with the square of d d. If y = 12 y = 12 when m = 3 m = 3, n = 8 n = 8, and d = 2, d = 2, find the constant k k, then use k k to find y y when m = −3 m = − 3, n = 18 n = 18, and d = 3 d = 3.

  21. Word Problems Involving Subtraction

    Word Problems with Subtraction How to solve word problems that involve subtraction? Examples: 1. At the beginning of the day, there were 339 tickets available for the raffle. By the end of the day, all but 54 tickets were sold. How many tickets were sold? 2. One month there were 6514 visitors at Harborton Aviary.

  22. 5 Strategies to Teach Multi-Step Word Problems: Teacher's Guide

    Strategies to Teach Multi-Step Word Problems. Now, let's delve into the 5 strategies that teachers can employ to effectively teach multi-step word problem-solving to their students. Model the Problem-Solving Process. Provide Clear Problem-Solving Strategies. Provide Scaffolded Practice.

  23. Teaching Methods for Solving Word Problems

    The video and article will provide you with two specific methods for solving word problems and ideas for teaching your students these skills. You will learn about common strategies required for solving word problems and challenges that arise so you can help correct and support your students.

  24. Mathematician who tamed randomness wins Abel Prize

    Michel Talagrand laid mathematical groundwork that has allowed others to tackle problems involving random processes.

  25. Add & subtract fractions word problems

    Like & unlike denominators. Below are our grade 5 math word problem worksheet on adding and subtracting fractions. The problems include both like and unlike denominators, and may include more than two terms. Worksheet #1 Worksheet #2 Worksheet #3 Worksheet #4. Worksheet #5 Worksheet #6.